File:  [Local Repository] / db / baza / izmi1718.txt
Revision 1.2: download - view: text, annotated - select for diffs - revision graph
Tue Oct 30 11:49:04 2018 UTC (5 years, 7 months ago) by rubashkin
Branches: MAIN
CVS tags: HEAD
*** empty log message ***

Чемпионат:
Синхронный турнир "Из Минска с любовью" - 2017/18

Дата:
13-Oct-2017 - 13-Apr-2018

Тур:
1 этап

Дата:
13-Oct-2017

Редактор:
1-18 - Юрий Разумов; 19-36 - Дмитрий Буценец (Могилёвцы)

Инфо:
Юрий Разумов благодарит Артема Гулецкого, Андрея Забавина, Марию
Кленницкую, Евгения Миротина, команду "Утро в Запретном лесу". Дмитрий
Буценец благодарит за тестирование и помощь в подготовке пакета: Михаила
Савченкова, Серафима Шибанова, Галину Пактовскую (все - Москва), Марию
Минакову (Санкт-Петербург), Евгения Миротина, Михаила Карпука, Юрия
Разумова, Артема Гулецкого, Рената Рустамова, Дарью Данилевич, Евгения
Лешковича, Даниила Шункевича, Сергея Дубелевича, а также команды
"Одушевленные аэросани" и "Мамонтлошечка" (все - Минск).

Вопрос 1:
Застенчивая героиня компьютерной игры не так давно перешла в новую
школу. Комментируя афишу на стене школы, она надеется, что ИКС никогда
не закончится. Какое русское слово мы заменили ИКСОМ?

Ответ:
Октябрь.

Комментарий:
Постер приглашал на празднование Хэллоуина.
   Начинаем октябрьский этап синхрона "Из Минска с любовью".

Источник:
Life is Strange, Episode 1.

Автор:
Юрий Разумов

Вопрос 2:
Алистер Кроули успешно создавал репутацию могущественного мага. Какими
двумя эпитетами называет Кроули российский литературовед Иван Мартов?

Ответ:
Великий и ужасный.

Комментарий:
То же делал Гудвин.

Источник:
https://gorky.media/context/parizh-eto-tsirtseya-prevrashhayushhaya-russkih-v-svinej/

Автор:
Антон Воронько

Вопрос 3:
Книга Пола Оффита об опасных научных открытиях называется "ЕЕ
лаборатория". Напишите ЕЕ имя.

Ответ:
Пандора.

Комментарий:
Слово "открытия" - подсказка.

Источник:
Paul A. Offit, Pandora's Lab: Seven Stories of Science Gone Wrong.

Автор:
Юрий Разумов

Вопрос 4:
Рост цен в одной отрасли нередко объясняют не только дорогими
исследованиями, но и большИм числом законов и правил. Блогер Скотт
Александр отмечает, что это вряд ли единственная причина, ведь ИХ
деятельность регулируется гораздо слабее, а цены растут так же быстро.
Назовите ИХ одним словом.

Ответ:
Ветеринары.

Комментарий:
Современная медицина (особенно в развитых странах) регулируется огромным
количеством законов, а разработка новых лекарств требует дорогостоящих
тестирований. Лечение животных не так зарегулировано, однако и там цены
растут.

Источник:
http://slatestarcodex.com/2017/02/09/considerations-on-cost-disease/

Автор:
Юрий Разумов

Вопрос 5:
В 1997 году на поверхность Марса был спущен марсоход "Sojourner"
[соджОрнер], оснащенный несколькими камерами. День высадки марсохода
веб-сайт НАСА называет днем, когда ОН остановился. Назовите ЕГО одним
словом.

Ответ:
Интернет.

Комментарий:
В 1997 году широкополосное соединение было редкостью, а фотографии,
сделанные марсоходом, хотели увидеть многие. Французскому правительству
пришлось прямо попросить граждан прекратить открывать сайт НАСА, чтобы
телефонная связь в стране заработала стабильнее.

Источник:
https://www.nasa.gov/specials/pathfinder20/

Автор:
Юрий Разумов

Вопрос 6:
Когда больному королю из пьесы Эжена Ионеско кажется, что ему стало
лучше, его жена возражает и говорит, что тот умрет через полтора часа, а
именно - ПРОПУСК. Заполните пропуск тремя словами.

Ответ:
В конце спектакля.

Зачет:
В конце пьесы; в конце книги; на последней странице.

Источник:
Э. Ионеско. Король умирает. http://flibusta.is/b/103768/read

Автор:
Кирилл Шкурин

Вопрос 7:
В недавней статье Пэт МОрроу отмечает, что летом в большинстве лиг
перерыв, поэтому ЕГО выход очень помог букмекерской конторе Морроу.
Назовите ЕГО точно.

Ответ:
Седьмой сезон "Игры престолов".

Зачет:
Новый сезон "Игры престолов"; очередной сезон "Игры престолов".

Комментарий:
Букмекеры стали принимать ставки на смерти персонажей, а также на то,
кто в итоге займет Железный трон. Этот вопрос неслучайно стоит после
вопроса о смерти короля.

Источник:
https://www.theringer.com/2017/7/20/16078298/tv-game-of-thrones-gambling-bovada-99918a2bf471

Автор:
Юрий Разумов

Вопрос 8:
Словами "ДЕЛАТЬ ЭТО" мы заменили другие слова.
   Историк ДИэрмэд МаккАлок предполагает, что взгляды Парацельса со
временем становились радикальнее потому, что он не мог иметь много книг,
ведь часто ДЕЛАЛ ЭТО. Кто ДЕЛАЕТ ЭТО в известной задаче?

Ответ:
Коммивояжер.

Комментарий:
Парацельс не очень интересовался мнением отцов церкви - частично потому,
что постоянно переезжал из города в город (нередко - из-за ухудшившихся
отношений с местными властями) и возил с собой очень мало книг.

Источник:
Diarmaid MacCulloch, Reformation: Europe's House Divided 1490-1700.

Автор:
Юрий Разумов

Вопрос 9:
Описывая след, оставляемый винтами ледокола, Дмитрий ЛОбусов упоминает
ЕГО. На Олимпиаде 2010 года половина медалей, разыгранных в НЕМ,
досталась спортсменам из США. Назовите ЕГО.

Ответ:
Хаф-пайп.

Зачет:
Хафпайп; half-pipe.

Комментарий:
Если вращать достаточно крепкий винт над поверхностью льда, получится
что-то очень похожее на хаф-пайп. Слово "половина" - подсказка.

Источник:
   1. https://topwar.ru/95774-sokrushayuschiy-ldy.html
   2. https://en.wikipedia.org/wiki/Snowboarding_at_the_2010_Winter_Olympics

Автор:
Сергей Алексетович

Вопрос 10:
В романе "Флатландия" описан двумерный мир, жители которого представляют
собой геометрические фигуры. В одной из глав романа рассказывается об
обычаях аристократов Флатландии. Какие два слова в предыдущем
предложении мы заменили словом "аристократов"?

Ответ:
Высших кругов.

Комментарий:
В романе Эдвина Эбботта общественное положение жителя Флатландии во
многом определяется количеством его углов. Высшие позиции в обществе
занимают круги.

Источник:
Э. Эбботт. Флатландия. http://flibusta.is/b/179946/read

Автор:
Кирилл Шкурин

Вопрос 11:
Полное имя писателя Эдвина Эбботта - Эдвин Эбботт Эбботт. Дело в том,
что его родители ПРОПУСК. Родители какого художника тоже ПРОПУСК?

Ответ:
[Анри де] Тулуз-Лотрека.

Комментарий:
Второе имя Эбботта - это девичья фамилия его матери. Она такая же, как
фамилия отца, потому что мать и отец были кузенами. То же справедливо и
для Тулуз-Лотрека, который, как считается, страдал от различных
заболеваний из-за кровосмесительной связи родителей.

Источник:
   1. https://ru.wikipedia.org/wiki/Эбботт,_Эдвин_Эбботт
   2. https://ru.wikipedia.org/wiki/Тулуз-Лотрек,_Анри_де

Автор:
Евгений Миротин

Вопрос 12:
Осмотрев заболевшего главного героя мультфильма "Джек и механика
сердца", специалист говорит, что сердце в полном порядке. Какие три
слова мы заменили словами "в полном порядке"?

Ответ:
Работает как часы.

Комментарий:
У героя мультфильма действительно часы вместо сломавшегося в детстве
сердца.

Источник:
Мультфильм "Джек и механика сердца" (2013), реж. Стефан Берла.

Автор:
Тимофей Прокопенко

Вопрос 13:
Дуплет.
   1. Героиня КрИстофа Хайна говорит, что ее монотонная жизнь движется
от весеннего события к осеннему и обратно. Назовите любое из этих
событий четырьмя словами.
   2. Подобную жизнь героиня КрИстофа Хайна сравнивает с ЕГО движением.
Назовите ЕГО одним словом.

Ответ:
   1. Переход на летнее время.
   2. Маятник.

Зачет:
   1. Переход на зимнее время.

Комментарий:
Жизнь героини настолько монотонна, что только перевод часов вносит в нее
какое-то разнообразие. Неудивительно, что эту метафору она дополняет
сравнением с маятником.

Источник:
Christoph Hein, Der Fremde Freund.

Автор:
Антон Воронько

Вопрос 14:
По ошибке купив поддельную картину Гойи, два испанца попытались продать
ее кому-то еще, однако вскоре после успешной сделки были арестованы.
Дело в том, что покупатель был ИМ. Назовите ЕГО словом с двумя корнями.

Ответ:
Фальшивомонетчик.

Комментарий:
Когда два брата, получив 1,7 миллиона швейцарских франков за картину,
попытались положить их на счет в банке, оказалось, что все эти деньги
такие же фальшивые, как и картина. Кроме того, братья еще до сделки
заплатили мошеннику 300 тысяч евро - он утверждал, что является только
агентом арабского шейха, и хотел получить процент от сделки.

Источник:
https://www.independent.co.uk/news/world/europe/con-mens-attempt-to-sell-forged-goya-painting-backfires-when-they-are-paid-with-fake-money-10063027.html

Автор:
Юрий Разумов

Вопрос 15:
Словами "ДЕЛАТЬ ЭТО" мы заменили другие слова.
   Героиня сериала готовится к трудному разговору с отцом и рассказывает
подруге, кАк в детстве отец наказал ее, застав курящей. Подруга в шутку
надеется, что отец не заставит героиню СДЕЛАТЬ ЭТО. Какой бог ДЕЛАЛ ЭТО?

Ответ:
Кронос.

Зачет:
Сатурн.

Комментарий:
Отец заставил героиню съесть сигарету. Своих детей, по легенде, съел
Кронос (он же Сатурн), что можно увидеть, например, на картине Гойи.

Источник:
   1. Телесериал "Друзья", s08e08, "The One With The Stripper".
   2. https://ru.wikipedia.org/wiki/Кронос

Автор:
Кирилл Шкурин

Вопрос 16:
Словом "ИКС" мы заменили другое слово.
   Отмечая юный возраст участников, Андрей Мовчан называет российские
протесты марта 2017 года "революцией школьных ИКСОВ". Персонажем какого
сериала является ИКС?

Ответ:
"Даша-путешественница".

Зачет:
"Dora the Explorer".

Комментарий:
Среди протестующих было много школьников и студентов, поэтому Мовчан
назвал акции "революцией рюкзаков". Рюкзак - один из главных героев
сериала о Даше-путешественнице.

Источник:
   1. https://republic.ru/posts/81172
   2. https://en.wikipedia.org/wiki/Dora_the_Explorer

Автор:
Антон Воронько

Вопрос 17:
Американский закон рекомендует финансовым компаниям обязать сотрудников
ДЕЛАТЬ ЭТО. В пример нередко приводят сотрудника банка ТошихИде ИгУчи,
который 11 лет не ДЕЛАЛ ЭТОГО, чтобы никто не заметил нелегальные сделки
на миллиарды долларов. Что такое "ДЕЛАТЬ ЭТО"?

Ответ:
Брать отпуск.

Зачет:
Уходить в отпуск; другие синонимичные глаголы.

Комментарий:
Сложные схемы мошенничества требуют постоянного контроля и присутствия,
поэтому финансовые регулирующие органы рекомендуют делать отпуска
обязательными или хотя бы присматривать за сотрудниками, которые не
пропускают ни одного рабочего дня.

Источник:
https://www.nytimes.com/1995/12/03/business/earning-it-workaholics-aren-t-the-only-ones-who-hate-vacations.html

Автор:
Юрий Разумов

Вопрос 18:
Последний вопрос первого тура.
   Герой романа Томаса Харди долго разглядывает трилобита, а также
отмечает неожиданно сильный ветер. Какой англоязычный термин появился
благодаря этой сцене?

Ответ:
Клиффхэнгер.

Комментарий:
Герой в прямом смысле висит на краю скалы, а потому может внимательно
рассмотреть окаменелость.

Источник:
   1. https://en.wikipedia.org/wiki/Cliffhanger
   2. Thomas Hardy, A Pair of Blue Eyes.

Автор:
Дарьяна Добрянская

Вопрос 19:
В одном сериале на двери в полицейском участке висит табличка, на
которой можно увидеть слово "disturb" [дистёрб] - "беспокоить". Что
изображено на табличке рядом с этим словом?

Ответ:
Пончик.

Зачет:
Donut; doughnut; дОнат. Незачет: Don't; do not.

Комментарий:
Слово "пончик" по-английски звучит как "donut" [дОнат] и созвучно с
отрицанием "не" в выражении "не беспокоить". Так Дэвид Линч в сериале
"Твин Пикс" обыгрывает известный стереотип о полицейских.

Источник:
   1. Телесериал "Твин Пикс", s03e03.
   2. http://welcometotwinpeaks.com/inspiration/donut-disturb-sign/

Автор:
Дарья Данилевич, Ренат Рустамов (Минск)

Вопрос 20:
В одной поэме гром без дождя назван ТАКИМ. Люди из касты БхАнту считают,
что ТАКАЯ женщина опасна для садов. Какое прилагательное мы заменили
словом "ТАКАЯ"?

Ответ:
Бесплодная.

Комментарий:
Примитивные племена часто сравнивают плодовитость природы с
плодовитостью женщины. В первой части вопроса речь идет о поэме Томаса
Элиота "Бесплодная земля".

Источник:
   1. Т. Элиот. Бесплодная земля. http://flibusta.is/b/282891/read
   2. С. Бовуар. Второй пол. http://flibusta.is/b/491900/read

Автор:
Дмитрий Буценец (Могилёвцы)

Вопрос 21:
Опасаясь исказить память о прошлом, герой КОрмака Маккарти старается
реже предаваться воспоминаниям. Аргументируя свою позицию, персонаж
приводит в пример игру. Какую?

Ответ:
"Испорченный телефон".

Зачет:
"Сломанный телефон".

Комментарий:
Персонаж считает, что каждое воспоминание наносит вред оригиналу, т.к.,
осознанно или нет, ты изменяешь то, что вспоминаешь.

Источник:
К. Маккарти. Дорога. http://flibusta.is/b/184846/read

Автор:
Дмитрий Буценец (Могилёвцы)

Вопрос 22:
Художник, разработавший карету для одного фильма, сравнил себя с НЕЙ
главной героини. В русском прокате фильм "Bella Mafia" [бЭлла мАфиа]
вышел под названием "ОНА". Назовите ЕЕ.

Ответ:
Крестная мать.

Зачет:
Крестная.

Комментарий:
Речь идет о фильме "Золушка", в котором, как и в сказке, фея-крестная
превращает тыкву в карету. Фильм "Крестная мать" снят на ту же тему, что
и "Крестный отец", но на 25 лет позже, и повествует о женщине, ставшей
главой мафии после убийства ее мужа.

Источник:
   1. https://ru.disney.wikia.com/wiki/Золушка_(2015)
   2. https://ru.wikipedia.org/wiki/Крёстная_мать_(фильм)

Автор:
Дмитрий Буценец (Могилёвцы), Никита Шевела (Минск)

Вопрос 23:
На верхнем уровне воронки продаж находятся потенциальные покупатели, а
на нижнем - успешные сделки. Если мАркетинг максимально эффективен, то
воронка СДЕЛАЕТ ЭТО. В одном фэнтези-романе описывается кролик, который
умеет ДЕЛАТЬ ЭТО. Что мы заменили словами "ДЕЛАТЬ ЭТО"?

Ответ:
Превращаться в цилиндр.

Зачет:
Трансформироваться в цилиндр; преобразовываться в цилиндр; становиться
цилиндром.

Комментарий:
В своем романе Джоан Роулинг подшучивает над классическим фокусом.
Воронка продаж - маркетинговый инструмент, который позволяет увеличить
количество клиентов, найти ошибки на каждой из стадий и устранить их, а
также привлечь целевую аудиторию. Чем эффективнее маркетинг - тем ближе
воронка к цилиндру.

Источник:
   1. https://marketer.ua/voronka-prodazh-ne-rabotaet/
   2. Дж.К. Роулинг. Гарри Поттер и узник Азкабана.
http://flibusta.is/b/265597/read

Автор:
Дмитрий Буценец (Могилёвцы)

Вопрос 24:
Персонаж одного рассказа сравнивает психику после нервного срыва с НЕЙ и
говорит, что даже после восстановления прежней надежности не будет.
Комментируя свой перфОрманс, Ай ВейвЕй заявил, что ОНА времен династии
Хань должна заставить общество пересмотреть отношение к искусству и к
прошлому. Назовите ЕЕ двумя словами.

Ответ:
Разбитая ваза.

Комментарий:
Фарфоровые вазы являются важным направлением искусства Древнего Китая.
Современный китайский художник Ай Вейвей известен своими перформансами
на грани вандализма. Так, художник приобрел, а затем разбил вазу
династии Хань.

Источник:
   1. С. Кинг. Иногда они возвращаются.
https://books.google.ru/books?id=qeG7AAAAQBAJ&pg=PT22#v=onepage&q&f=false
   2. http://be-inart.com/post/view/1733

Автор:
Дмитрий Буценец (Могилёвцы)

Вопрос 25:
(pic: 20170877.jpg)
   Многие гены выполняют небольшую работу в функционировании
биохимических путей, хотя в случае мутаций могут быть виновниками
различных нарушений. Олег Рева называет эти гены представителями
устаревающей профессии. Какой?

Ответ:
Стрелочник.

Комментарий:
Рева говорит, что многие гены - всего лишь стрелочники на перекрестках
биохимических путей, и роль у них сравнительно скромная. Несмотря на то
что мутации, как правило, вызваны внешними факторами, виноват, как
известно, стрелочник.

Источник:
М. Ридли. Геном (в переводе Олега Ревы).
http://flibusta.is/b/349905/read

Автор:
Дмитрий Буценец (Могилёвцы)

Вопрос 26:
Некоторые из противников теории англичанина поддерживали аристотелевское
учение о "приливе и отливе". Когда в 1661 году Марчелло увидел связь
между ПЕРВЫМИ и ВТОРЫМИ, остававшиеся споры вокруг теории прекратились.
Назовите ПЕРВЫЕ и ВТОРЫЕ.

Ответ:
Артерии, вены.

Зачет:
В любом порядке.

Комментарий:
Идеи ГАрвея о кровообращении прошли через шквал критики. Ранее ученые
придерживались теории ГалЕна, согласно которой кровь образуется в
печени, а некоторые - и еще более древних теорий. Несмотря на то что
многие приняли идеи Гарвея еще при его жизни, убедительные
доказательства появились через четыре года после смерти ученого, когда
МальпИги, воспользовавшись микроскопом, обнаружил капиллярную сеть,
соединяющую артерии и вены.

Источник:
   1. М. Энгельгардт. Уильям Гарвей. Его жизнь и научная деятельность.
https://profilib.com/chtenie/58198/mikhail-engelgardt-uilyam-garvey-ego-zhizn-i-nauchnaya-deyatelnost-7.php
   2. С.Н. Зигуненко. 100 великих тайн медицины.
http://flibusta.is/b/482131/read

Автор:
Дмитрий Буценец (Могилёвцы)

Вопрос 27:
Считается, что ЕГО традиционно поддерживают представители среднего и
высшего классов. Так, одно из ЕГО прозвищ связано с пирожными, которые в
начале XX века были доступны лишь зажиточным горожанам. Назовите ЕГО.

Ответ:
[Футбольный клуб] "Реал Мадрид".

Комментарий:
"Реал" в отношении "фанатского слоя" действительно можно назвать
"королевским" клубом. Прозвище "Сливочные", закрепившееся в русском
языке, могло вам намекнуть на крем. Впрочем, на самом деле это очень
вольное переложение испанского "Los Merengues" [лос мерЕнгес]. Меренги,
они же безе, имеют "сливочный" цвет, чем, видимо, и обусловлена русская
локализация прозвища.

Источник:
https://ru.wikipedia.org/wiki/Мадридское_дерби

Автор:
Ренат Рустамов (Минск)

Вопрос 28:
(pic: 20170878.jpg)
   Перед вами фрагмент иллюстрации к одной статье. Ее автор пишет, что
именно так в XIX веке должен был выглядеть "ОН", чтобы избежать
скандала. Назовите "ЕГО".

Ответ:
"Завтрак на траве".

Комментарий:
"Завтрак на траве" - скандально известная картина Эдуарда Мане. Выбор
сюжета картины - двое полностью одетых мужчин с нагой женщиной на
природе - вызвал полное непонимание и обвинения в плохом вкусе.

Источник:
https://ru.wikipedia.org/wiki/Завтрак_на_траве_(картина_Мане)

Автор:
Дмитрий Буценец (Могилёвцы)

Вопрос 29:
В одном эссе Сергей Довлатов выделяет следующие последовательные уровни:
сАбвей, супермаркет, TV [ти-ви], дейли ньюз, чайна таун. Какое слово мы
пропустили в названии каждого уровня?

Ответ:
Инглиш.

Зачет:
English; английский.

Комментарий:
Сергей Довлатов, уехавший в США по политическим причинам, рассказывает
об уровнях владения английским языком эмигрантами. Причем самым высшим
уровнем он называет понимание английского, на котором говорят другие
эмигранты, в частности китайцы.

Источник:
С.Д. Довлатов. Блеск и нищета русской литературы.
http://flibusta.is/b/129387/read

Автор:
Дарья Данилевич (Минск)

Вопрос 30:
В свое время массовые репрессии в Латинской Америке были настолько
обычным делом, что глагол "исчезнуть" стал ТАКИМ. По современным данным
одной из причин проявлений ТАКОГО ЕГО является активное размножение
нейронов. Что мы заменили словами "ТАКОЙ ОН"?

Ответ:
Переходный возраст.

Комментарий:
В испанском языке глагол "исчезнуть" стал переходным, т.е. начали
говорить "исчезнуть кого-либо" в значении "устранить, уничтожить". Ранее
ученые считали, что активное размножение нейронов происходит только в
раннем детском возрасте, однако современные исследования показывают, что
повторная волна размножения нейронов происходит в подростковом возрасте,
чем отчасти обусловлено не всегда адекватное поведение в этот период.

Источник:
   1. М. Хем. Быть диктатором. Практическое руководство.
http://flibusta.is/b/457718/read
   2. https://www.infoniac.ru/news/10-faktov-kotoryh-kazhdyi-roditel-dolzhen-znat-o-mozge-svoego-podrostka.html

Автор:
Никита Шевела (Минск), Дмитрий Буценец (Могилёвцы)

Вопрос 31:
(pic: 20170879.jpg)
   Перед вами картина, изображающая игру, популярную на рубеже XVIII-XIX
веков. Согласно одной версии, поднятые руки девушек символизируют ЕЕ.
Назовите ЕЕ.

Ответ:
Гильотина.

Комментарий:
Изображенная игра напоминает игру "ручеек": на последней строчке руки
опускаются, и тот участник, который оказывается пойман, - выбывает.
Согласно мрачной теории, игра была полна символизма и отсылала к модному
в то время способу казни.

Источник:
   1. http://academics.hamilton.edu/mediascholarship/internproject/steph.html
   2. https://en.wikipedia.org/wiki/File:Agnes_Rose_Bouvier00.jpg

Автор:
Дмитрий Буценец (Могилёвцы)

Вопрос 32:
Описывая выступления Шарля де Голля, один еженедельник писал, что лица
людей, которые воодушевленно поддерживали генерала, становились такого
же цвета, как ОН. Чтобы избежать кровотечения в бою, ЕГО, как правило,
отрезают. Назовите ЕГО двумя словами.

Ответ:
Петушиный гребень.

Зачет:
Петушиный гребешок; гребень петуха; гребешок петушка.

Комментарий:
Бойцовских петухов готовят заранее: так, в возрасте полугода им обычно
отрезают гребень. Рассказывая о патриотизме и делая отсылку к
национальному символу Франции, репортер сравнил раскрасневшиеся лица
людей с петушиным гребнем.
   z-checkdb: Это ошибка переводчика: в оригинале упоминается цвет мака
(coquelicot), см.
https://archive.org/details/LeDeuxiemeSexeTome2SimoneDeBeauvoir/page/n667
(Евгений Рубашкин).

Источник:
   1. https://ru.wikipedia.org/wiki/Петушиные_бои
   2. http://allrefrs.ru/2-20416.html

Автор:
Дмитрий Буценец (Могилёвцы)

Вопрос 33:
Для изготовления особенно прочных механизмов нацистская Германия в
большом количестве покупала редкое сырье. Сэм Кин образно замечает, что
поставки сырья удовлетворяли ЕГО немецкой промышленности. Назовите ЕГО
устойчивым выражением.

Ответ:
Волчий аппетит.

Зачет:
Волчий голод.

Комментарий:
Речь идет о химическом элементе - вольфраме. Как известно, сплавы
вольфрама обладают высокой тугоплавкостью и прочностью. Слово "вольфрам"
можно перевести с немецкого языка как "волчья пена". Кстати, Салазар,
который продавал Германии вольфрам, нажил на нем немалое богатство.

Источник:
С. Кин. Исчезающая ложка, или Удивительные истории из жизни
периодической таблицы Менделеева. http://flibusta.is/b/429981/read

Автор:
Дмитрий Буценец (Могилёвцы)

Вопрос 34:
[Ведущему: во втором предложении сделать паузу между словами "ТАКОГО" и
"ОНО".]
   В фильме о тяжелой жизни рыбаков говорится, что ОНО ТАКОЕ. У ТАКОГО
ОНО ловит стрелы молний. Что мы заменили словами "ОНО ТАКОЕ"?

Ответ:
Море горькое.

Комментарий:
Говоря об изматывающем труде рыбаков и их частой гибели, героиня
печально размышляет, что море горькое. В "Песне о Буревестнике" Горького
море "ловит и гасит" молнии.

Источник:
   1. Х/ф "Земля дрожит" (1948), реж. Лукино Висконти.
http://cinematext.ru/movie/zemlja-drozhit-la-terra-trema-episodio-del-mare-1948/
   2. https://murzim.ru/nauka/himiya/21084-soli-v-prirodnoy-vode.html
   3. http://gorkiy-lit.ru/gorkiy/proza/rasskaz/pesnya-o-burevestnike.htm

Автор:
Дмитрий Буценец (Могилёвцы)

Вопрос 35:
[Ведущему: четко произнести букву "д" в слове "Лад".]
   В переводе одного романа описывается высокотехнологичный в прошлом
город Лад, разрушенный его жителями. После разъяснения автора в более
поздних переводах название города немного изменили. На какое?

Ответ:
Луд.

Зачет:
Лудд.

Комментарий:
Стивен Кинг рассказал, что название города произошло от названия
противников промышленной революции - луддитов. Луддиты считали своим
предводителем некоего Неда Лудда, которому приписывалось уничтожение
двух чулочных станков. Подобно луддитам жители из произведения Кинга
разрушали свой город.

Источник:
https://ru.wikipedia.org/wiki/Бесплодные_земли

Автор:
Дмитрий Буценец (Могилёвцы)

Вопрос 36:
   <раздатка>
   Новое кино
   </раздатка>
   На розданном вам материале часть цитаты главной фигуры движения
"Новое кино" ГлАубера РОши. Какими двумя короткими словами она
заканчивается?

Ответ:
Это я.

Комментарий:
Легендарная фраза "Государство - это я" приписывается Людовику XIV.

Источник:
https://en.wikipedia.org/wiki/Glauber_Rocha

Автор:
Ренат Рустамов (Минск)

Тур:
2 этап

Дата:
10-Nov-2017

Редактор:
1-18 - Евгений Лешкович (Минск); 19-36 - Валерий Семёнов (Минск -
Могилев)

Инфо:
Евгений Лешкович благодарит за тестирование и помощь в подготовке пакета
Андрея Танану, Анну Якушевич, Вадима Кузмича, Даниила Шункевича,
Александра Шустера, Валерия Семёнова, Дмитрия Буценца, Анастасию
Балмакову, Василия Бобкова, Михаила Карпука, Александру Ермалович, Елену
Ваксман-Атрохову, Дарью Соловей, Руслана Огородника, Екатерину Лагуту,
Андрея Кравченко, Игоря Тюнькина, Никиту Лопуха, Сергея Очтова, а также
команды "И тебя тоже" и "Жидкость для розжига". Валерий Семёнов
благодарит за помощь в работе над пакетом всех авторов вопросов, а также
команды "И тебя тоже", "Жидкость для розжига" и "Одушевленные аэросани"
за тестирование и ценные замечания.

Вопрос 1:
Говоря об ИКСЕ своего друга, герой романа Ирвина Уэлша использует
выражения "паук брюхом" и "скорпион хвостом". Какие два слова,
начинающиеся на одну и ту же букву, мы заменили ИКСОМ?

Ответ:
Плохой почерк.

Комментарий:
Обычно такой почерк характеризуют как "курица лапой", но в данном случае
всё так плохо, что герой решил поискать другие сравнения.
   Игровое жюри надеется, что ему их искать не придется.

Источник:
И. Уэлш. Порно. http://flibusta.is/b/77708/read

Автор:
Евгений Лешкович (Минск)

Вопрос 2:
По мнению МИкала Хема, у африканских диктаторов есть одно отличие от
остальных - они не любят покупать роскошные автомобили. В качестве
причины Хем называет ИХ. В известном выражении ОНИ упоминаются в паре...
С кем?

Ответ:
С дураками.

Комментарий:
Африканские диктаторы не покупают дорогих автомобилей, потому что в их
странах плохие дороги и кататься всё равно негде. Дураки и дороги - две
вечные проблемы России.

Источник:
М. Хем. Быть диктатором. Практическое руководство.
http://flibusta.is/b/457718/read

Автор:
Евгений Лешкович (Минск)

Вопрос 3:
(pic: 20170880.jpg)
   Перед вами дом, в названии которого есть ОНА. Где ОНА находится в
заглавии произведения второй половины XX века?

Ответ:
На склоне.

Комментарий:
ОНА - улитка. Речь в вопросе идет о произведении Аркадия и Бориса
Стругацких.

Источник:
   1. https://novate.ru/blogs/071109/13373/
   2. https://ru.wikipedia.org/wiki/Улитка_на_склоне

Автор:
Вадим Кузмич (Логойск)

Вопрос 4:
Историк Сергей Иванов не любит вопросов о будущем. По мнению Иванова,
задавать историку такие вопросы - то же самое, что спрашивать ИКСА о
бессмертии души. ИКСОМ можно назвать героя произведения 1632 года.
Назовите автора этого произведения.

Ответ:
Рембрандт [Харменс ван Рейн].

Комментарий:
По мнению Сергея Иванова, спросить историка о будущем - то же самое, что
спросить патологоанатома о бессмертии души. Заглавный герой картины
Рембрандта "Урок анатомии доктора Тульпа" был в том числе и
патологоанатомом.

Источник:
   1. https://arzamas.academy/materials/191
   2. https://ru.wikipedia.org/wiki/Урок_анатомии_доктора_Тульпа

Автор:
Евгений Лешкович (Минск)

Вопрос 5:
Герой американского мультсериала говорит, что смотрит в будущее, и носит
значок с надписью "Иван 2028". Какие две буквы мы пропустили?

Ответ:
ка.

Комментарий:
Полностью надпись на значке выглядит как "Иванка 2028". Видимо, персонаж
не сомневается, что одним из кандидатов в президенты США в 2028 году
будет Иванка Трамп. Это хорошо впишется в американский политический
тренд, заданный семействами Бушей и Клинтонов.

Источник:
https://pikabu.ru/story/simpsonyi_prodolzhayut_prorochit_4616031

Автор:
Андрей Танана, Анна Якушевич (Минск)

Вопрос 6:
После второго срока Джордж Вашингтон поселился в поместье МАунт-ВЕрнон.
Вскоре он распорядился, чтобы вокруг поместья были установлены
неправильные указатели. Современный журналист, рассказывающий об этой
истории, упоминает слово с удвоенной согласной. Какое?

Ответ:
Папарацци.

Комментарий:
Вашингтона очень быстро утомили назойливые журналисты. Пришлось сделать
так, чтобы случайный человек никак не мог добраться до Маунт-Вернона.

Источник:
"Дилетант", 2017, N 1. - С. 21.

Автор:
Евгений Лешкович (Минск)

Вопрос 7:
В стихотворении "Танки идут по Праге" скрепки советских чиновников
превращаются в НИХ. Стадия ЕЕ может длиться от нескольких недель до
нескольких лет. Назовите ЕЕ.

Ответ:
Гусеница.

Комментарий:
   "Танки идут по склепам,
   По тем, что еще не родились.
   Четки чиновничьих скрепок
   В гусеницы превратились".

Источник:
   1. http://www.ruthenia.ru/60s/evtushenko/tanki.htm
   2. https://ru.wikipedia.org/wiki/Гусеница

Автор:
Максим Осмоловский (Минск)

Вопрос 8:
У Аристофана рассказывается о встрече Эсхила и Еврипида. Упоминая об
этом, Михаил ШвыдкОй шутит, что ИХ история насчитывает уже двадцать пять
веков. Назовите ИХ словом, пишущимся через дефис.

Ответ:
Рэп-баттлы.

Комментарий:
Эсхил и Еврипид в пьесе "Лягушки" устроили жесткий поэтический поединок,
напомнивший Швыдкому современные рэп-баттлы. Кстати, победил Эсхил.

Источник:
https://meduza.io/shapito/2017/09/17/rep-battly-imeyut-davnyuyu-istoriyu-esli-otkryt-aristofana-mihail-shvydkoy-pozval-gnoynogo-na-telekanal-kultura

Автор:
Евгений Лешкович (Минск)

Вопрос 9:
Джеймс УиттАкер осуждает тестировщиков, склонных преувеличивать важность
каждого дефекта. Дальше Уиттакер проводит параллель с известной
историей, заменяя словом "баг" другое слово. Назовите это слово.

Ответ:
Волк.

Комментарий:
К тестировщику-паникеру со временем перестают прислушиваться. Уиттакер
сравнивает его с мальчиком, который кричал "Волки!".

Источник:
Дж. Уиттакер, Дж. Арбон, Дж. Кароло. Как тестируют в Google.
http://flibusta.is/b/382371/read

Автор:
Евгений Лешкович (Минск)

Вопрос 10:
Историк Маргарет Макмиллан пишет, что в начале XX века отношения
Великобритании с Соединенными Штатами тоже стали радушными. Какое слово
мы заменили в этом вопросе?

Ответ:
Сердечными.

Комментарий:
В это же время Великобритания вошла в состав Антанты. "Entente cordiale"
[антАнт кордиАль] означает "сердечное согласие". Слова "радушный" и
"сердечный" являются синонимами.

Источник:
М. Макмиллан. Война, которая покончила с миром. Кто и почему развязал
Первую мировую. http://flibusta.is/b/452488/read

Автор:
Евгений Лешкович (Минск)

Вопрос 11:
[Ведущему: как можно четче прочитать фамилию Квонг!]
   Выступавший в середине тридцатых хоккеист Ларри Квонг отличался
развитой мускулатурой и агрессивным стилем игры. Партнеры дали ему
прозвище, лишь одной буквой отличающееся от названия произведения.
Назовите это произведение.

Ответ:
"Кинг-Конг".

Комментарий:
Мощный и агрессивный хоккеист удостоился прозвища "Кинг-Квонг". Фильм
"Кинг-Конг" как раз вышел в середине тридцатых.

Источник:
https://www.sports.ru/tribuna/blogs/antarktida/1409410.html

Автор:
Евгений Лешкович (Минск)

Вопрос 12:
Главного героя анимационного фильма зовут Джек Фрост. Однажды его сестра
провалилась под лед, но Джек спас ее ценой собственной жизни. Кто должен
был озвучивать Джека по первоначальному плану режиссера?

Ответ:
Леонардо ди Каприо.

Комментарий:
История со спасением сестры сильно перекликается с сюжетом "Титаника",
главную роль в котором сыграл Ди Каприо. Кстати, там его персонажа тоже
зовут Джек.

Источник:
   1. https://ru.wikipedia.org/wiki/Хранители_снов
   2. http://www.kinobusiness.com/movies/Rise-of-the-Guardians/

Автор:
Анна Якушевич (Минск)

Вопрос 13:
Джованни Баттиста ГрАсси был одним из крупнейших исследователей малярии.
Автор книги о Грасси назвал ученого ИКСОМ ИГРЕКОВ. Одна из рецензий на
"ИКСА АЛЬФ" называется "Детский апокалипсис". Какое слово мы заменили
АЛЬФОЙ?

Ответ:
Муха.

Комментарий:
Переносчиками малярии являются комары. Грасси изучил их настолько
хорошо, что его прозвали "повелителем комаров". Книга "Повелитель мух"
повествует о локальном "детском апокалипсисе".

Источник:
   1. П. де Крюи. Охотники за микробами.
http://flibusta.is/b/511064/read
   2. https://www.kinopoisk.ru/user/737002/comment/1561680/

Автор:
Андрей Танана (Минск)

Вопрос 14:
Какао-бобы содержат вещества, способные увеличить склонность человека к
насилию. В какой стране было изобретено коншИрование - методика,
благодаря которой из какао-бобов удаляется до 80% таких веществ?

Ответ:
Швейцария.

Комментарий:
Чистые какао-бобы провоцируют нездоровую агрессию. Однако благодаря
коншированию они становятся абсолютно безобидными. Видимо, неспроста эта
методика была изобретена в стране, известной миролюбием и нейтралитетом!

Источник:
Т. Нилон. Битвы за еду и войны культур: тайные двигатели истории.
http://flibusta.is/b/482946/read

Автор:
Евгений Лешкович (Минск)

Вопрос 15:
В 1668 году во Франции началась эпидемия чумы. Население некоторых
городов сократилось наполовину, однако Париж при этом не пострадал.
Объясняя этот факт, Том Нилон пишет, что двумя годами ранее в Париже
началась мода на ад. Какие пять букв мы пропустили в вопросе?

Ответ:
лимон.

Комментарий:
По мнению Нилона, парижан от чумы спасла любовь к лимонаду. Цедра лимона
содержит вещества, убивающие переносчиков чумы. Цедру выбрасывали на
свалки, ее ели крысы, и это помогало естественным образом
продезинфицировать город.

Источник:
Т. Нилон. Битвы за еду и войны культур: тайные двигатели истории.
http://flibusta.is/b/482946.

Автор:
Евгений Лешкович (Минск)

Вопрос 16:
Корабль "Дредноут" отличался необычайной скоростью и нес на борту десять
12-дюймовых орудий. Рассказывая о "Дредноуте", современный историк
использует фразу, которую приписывают уроженцу Луисвилля. Назовите этого
уроженца.

Ответ:
Мохаммед Али.

Зачет:
Кассиус Клей.

Комментарий:
Корабль был быстрым и мощным, т.е. мог "порхать, как бабочка, и жалить,
как пчела". Эту фразу приписывают знаменитому боксеру Мохаммеду Али,
который родился в Луисвилле, штат Кентукки.

Источник:
   1. М. Макмиллан. Война, которая покончила с миром. Кто и почему
развязал Первую мировую. http://flibusta.is/b/452488/read
   2. http://mirfactov.com/10-mudrostey-mohammeda-ali/

Автор:
Евгений Лешкович (Минск)

Вопрос 17:
Емельян Пугачёв обращался к народу от имени императора и обещал отменить
крепостное право. Чтобы остановить самозванца, Екатерина II объявила,
что силу закона на территории России теперь будут иметь только ТАКИЕ
документы. Кто создал первый ТАКОЙ документ в Пфальце?

Ответ:
Иоганн Гутенберг.

Комментарий:
Печатного станка у пугачёвцев не было, поэтому приказ Екатерины II
считать законными только напечатанные манифесты лишил их способа
воздействовать на народ. Изобретатель книгопечатания Иоганн Гутенберг
проживал в княжестве Пфальц. Редактор посвящает вопрос 500-летию
белорусского книгопечатания.

Источник:
   1. А.В. Иванов. Вилы.
https://books.google.ru/books?id=TTlCDQAAQBAJ&pg=PT79#v=onepage&q&f=false
   2. https://ru.wikipedia.org/wiki/Гутенберг,_Иоганн
   3. https://ru.wikipedia.org/wiki/Майнц

Автор:
Евгений Лешкович (Минск)

Вопрос 18:
Полковник Дэвид Хэкворт называет себя счастливчиком. В статье Википедии
о Хэкворте упоминаются восемь сердец. Какое слово мы пропустили?

Ответ:
Пурпурных.

Комментарий:
Медаль "Пурпурное сердце" вручается военнослужащим, получившим ранения.
Хэкворт был ранен восемь раз, но всё же остался жив - настоящий
счастливчик!

Источник:
https://ru.wikipedia.org/wiki/Хэкворт,_Дэвид_Хаскелл

Автор:
Евгений Лешкович (Минск)

Вопрос 19:
Герой Диккенса называет недорогие лондонские трактиры географическими,
так как каждая АЛЬФА напоминает ему карту мира с указанием морских
течений. Необычная разновидность АЛЬФЫ описана в фольклоре. Назовите
АЛЬФУ одним словом.

Ответ:
Скатерть.

Комментарий:
На скатерти в недорогом трактире - следы от кружек и пищи.

Источник:
Ч. Диккенс. Большие надежды. http://flibusta.is/b/301012/read

Автор:
Ольга Ярошенко (Могилев)

Вопрос 20:
(pic: 20170881.jpg)
   Перед вами автопортрет Доменико ДурАнте, туринского художника первой
половины XX века. За свою карьеру он получил множество наград, а в 1905
году даже стал победителем... Чего?

Ответ:
Чемпионата Италии по футболу.

Комментарий:
Дуранте совмещал две профессии и успел провести 30 игр за туринский
"Ювентус", в футболке которого он себя и изобразил.

Источник:
https://ru.wikipedia.org/wiki/Дуранте,_Доменико

Автор:
Максим Корнеевец (Минск)

Вопрос 21:
Художник Владимир Маковский создал свою картину в конце XIX века. Более
известная картина на тот же сюжет была создана тридцатью с лишним годами
ранее, хотя разницу выдает только фасон платья. Назовите автора более
известной картины.

Ответ:
[Василий] Пукирев.

Комментарий:
На картине Пукирева "Неравный брак", написанной в 1862 году, у невесты
большой громоздкий кринолин, а на картине "К венцу" 1894 года у невесты
юбка значительно сужена. Гораздо более выражена разница в возрасте
героев обеих картин.

Источник:
http://www.liveinternet.ru/users/4231626/post421512623/

Автор:
Юлия Ткачёва (Минск)

Вопрос 22:
Героиня фантастического рассказа везет в Африку новый вирус, который
позволяет любому человеку осуществлять процесс фотосинтеза. При этом она
говорит, что у нее в сумке находится смерть одного из ИКСОВ. Назовите
ИКСОВ двумя словами.

Ответ:
Всадники Апокалипсиса.

Комментарий:
Человек, умеющий осуществлять процесс фотосинтеза, не зависит от
источников питания, поэтому Голод ему не страшен.

Источник:
У.Й. Уильямс. Зеленая Леопардовая Чума. http://flibusta.is/b/119581/read

Автор:
Ольга Ярошенко (Могилев)

Вопрос 23:
Чтобы продолжать наступление в битве под Москвой, немцы зачастую
разводили костры под НИМИ. Назовите ИХ двумя словами, начинающимися на
парные согласные.

Ответ:
Двигатели танков.

Зачет:
Дно танков; днища танков.

Комментарий:
Таким образом приходилось отогревать замерзшее топливо.

Источник:
М. Хейстингс. Вторая мировая война: Ад на земле.
http://flibusta.is/b/389254/read

Автор:
Валерий Семёнов (Минск)

Вопрос 24:
В 1926 году в московском Клубе радиолюбителей состоялся концерт,
вызвавший большой интерес не только у радиотехников, но и у
профессиональных музыкантов. Кто давал этот концерт?

Ответ:
[Лев] Термен.

Комментарий:
Инженер играл на придуманном им терменвоксе. Радиотехников интересовал
принцип работы, а музыкантов - новый инструмент.

Источник:
Б.Е. Черток. Ракеты и люди. Фили-Подлипки-Тюратам.
http://flibusta.is/b/82011/read

Автор:
Валерий Семёнов (Минск)

Вопрос 25:
Корней Чуковский рассказывает, что в 1924 году из помещения одной
биостанции стали систематически пропадать банки с биоматериалами.
Назовите основную причину пропаж двумя словами, начинающимися на парные
согласные.

Ответ:
Сухой закон.

Комментарий:
Земноводные были заспиртованными, и пропадали даже несмотря на то, что
спирт был с формалином.

Источник:
Н.Б. Лебина. Советская повседневность: нормы и аномалии. От военного
коммунизма к большому стилю. http://flibusta.is/b/415924/read

Автор:
Валерий Семёнов (Минск)

Вопрос 26:
В 20-х годах XIX века мастер Рьессек создал первый ИКС, добавив к
привычному устройству чернильную иглу. ИКС - это жанр средневековой
литературы, содержащий попытки систематизации истории мира. Какое слово
греческого происхождения мы заменили ИКСОМ?

Ответ:
Хронограф.

Комментарий:
Механизм Рьессека фиксировал измеренный промежуток времени с помощью
чернильной иглы, ставившей точку на циферблате. В хронографах отмечаются
одни из первых попыток ведения исторических летописей в хронологическом
порядке.

Источник:
   1. http://mywatch.ru/watch-art/art_1644.html
   2. https://ru.wikipedia.org/wiki/Хронограф
   3. https://ru.wikipedia.org/wiki/Хронограф_(книга)

Автор:
Андрей Цмыг (Минск)

Вопрос 27:
Герои современного рассказа спасаются от грозы во флорентийской базилике
ди Санта-Кроче, однако им кажется, что молнии оказались и там. Назовите
причину этого сложным словом.

Ответ:
Фотовспышки.

Зачет:
Фотоаппарат; фотограф.

Комментарий:
Базилика является популярным туристическим местом уже хотя бы в силу
того, что там находится усыпальница Галилея.

Источник:
У.Й. Уильямс. Зеленая Леопардовая Чума. http://flibusta.is/b/119581/read

Автор:
Ольга Ярошенко (Могилев)

Вопрос 28:
В этом вопросе ИКС заменяет одно слово.
   Цитата из Жюля Верна: "Взгляните-ка на ИКС, <...> разве это не живое
существо? Порой гневное, порой нежное! Ночью он спал, как и мы, и вот
просыпается в добром расположении духа после покойного сна!". Другой
фантаст говорил, что ИКС из его романа должен выглядеть как лавовая
лампа. Назовите этого другого фантаста.

Ответ:
[Станислав] Лем.

Комментарий:
ИКС - океан. Жюль Верн много писал на морскую тематику. Приведенная
цитата - из романа "Двадцать тысяч лье под водой". Океан из романа Лема
"Солярис" и впрямь был живым.

Источник:
   1. Ж. Верн. Двадцать тысяч лье под водой.
http://flibusta.is/b/399777/read
   2. https://pl.wikipedia.org/wiki/Solaris_(powie%C5%9B%C4%87)

Автор:
Павел Дернович (Минск)

Вопрос 29:
Внимание, в вопросе есть замена.
   В боевике под названием "ГАММА" главный герой наносит ракетный удар
по вулкану. Неграмотный школьник описал ГАММУ как разновидность японских
мультиков. Что мы заменили ГАММОЙ?

Ответ:
Магма.

Комментарий:
По сюжету фильма "Магма" главный герой спасает город от судьбы Помпеев,
запечатав жерло вулкана ударом ракеты. Школьник перепутал слова "магма"
и "манга". "Гамма" и "магма" - анаграммы, чем и обусловлена замена.

Источник:
   1. https://www.kinopoisk.ru/film/magma-2005-353592/
   2. "Наука и жизнь", 2016, N 9. - С. 89.

Автор:
Евгений Гацак (Минск)

Вопрос 30:
Легкодоступный набор рукописных изображений цифр MNIST часто
используется для проведения экспериментов в сфере машинного обучения.
Джеффри Хинтон назвал MNIST ЕЮ машинного обучения. Назовите ЕЕ словом
греческого происхождения.

Ответ:
Дрозофила.

Комментарий:
Благодаря простоте, удобству использования и доступности дрозофила уже
много лет является одним из самых популярных объектов экспериментов в
биологии.

Источник:
http://www.deeplearningbook.org/contents/intro.html

Автор:
Даниил Мальцев (Минск)

Вопрос 31:
(pic: 20170882.jpg)
   Перед вами бокалы для использования ТАМ. Кто упоминается ТАМ в
названии романа 2002 года?

Ответ:
Кафка.

Комментарий:
Заостренные ножки удобно втыкать в песок. "Кафка на пляже" - роман
Харуки Мураками.

Источник:
   1. http://www.etoday.ru/2016/07/bokaly-dlya-vina-dlya-romatycheskih-svidaniy-na-plyazhe.php
   2. https://ru.wikipedia.org/wiki/Кафка_на_пляже

Автор:
Максим Корнеевец (Минск)

Вопрос 32:
В первых вариантах известной игры игрокам предлагалось пользоваться
подручными предметами, поэтому в современном классическом наборе есть
ОН. Назовите ЕГО.

Ответ:
Наперсток.

Комментарий:
В первых вариантах монополии не было фишек, и игроки находили подручные
предметы. Наперсток очень удобен в качестве фишки, поэтому он есть в
списке традиционных фишек для игры в монополию. В классическом наборе
"Монополии" используются фишки в виде ретроавтомобиля, шляпы, тачки,
пса, корабля, ботинка, кошки и наперстка.

Источник:
   1. https://republic.ru/posts/83164
   2. https://lenta.ru/news/2017/02/17/monoliya/

Автор:
Валерий Семёнов (Минск)

Вопрос 33:
В 1765 году российскому историку Герарду Миллеру, скопившему широчайшую
коллекцию рукописей, выделили из казны огромную сумму в шесть тысяч
рублей на покупку ИКСА в Москве. Обладателем ИКСА был предусмотрительный
герой известного произведения. Назовите ИКС двумя словами.

Ответ:
Каменный дом.

Комментарий:
Пожары в тогда еще деревянной Москве были частым делом. Было
недопустимо, чтобы сгорели драгоценные источники русской истории.

Источник:
Артем Ефимов. С чего мы взяли. Три века попыток понять Россию умом. -
М.: Индивидуум паблишинг, 2017.

Автор:
Валерий Семёнов (Минск)

Вопрос 34:
В пещере Атапуэрке были найдены несколько скелетов гейдельбергского
человека. Правда, кости были раздроблены и сильно перемешаны. Тем не
менее, Станислав Дробышевский с оптимизмом говорит, что испанские
антропологи - это не ПЕРВАЯ и не ВТОРАЯ. Назовите ПЕРВУЮ и ВТОРУЮ двумя
словами, первое из которых одинаковое.

Ответ:
Королевская конница и королевская рать.

Комментарий:
Ученые всё же смогли эти скелеты собрать.

Источник:
С.В. Дробышевский. Достающее звено. Книга вторая: Люди.
http://flibusta.is/b/491954/read

Автор:
Валерий Семёнов (Минск)

Вопрос 35:
(pic: 20170883.jpg)
   Перед вами фотография, сделанная жившим на рубеже XIX и XX веков
изобретателем Этьеном-Жюлем Маре. Одна из его книг была в доме епископа.
Назовите фамилию этого епископа.

Ответ:
Райт.

Комментарий:
В книге Маре "Animal Mechanism" описаны механизмы птичьих полетов. После
прочтения этой книги его сыновья заинтересовались полетами. Известно,
что отец братьев Райт был священником.

Источник:
   1. Д. Маккалоу. Братья Райт. Люди, которые научили мир летать.
http://flibusta.is/b/480043/read
   2. https://ru.wikipedia.org/wiki/Маре,_Этьен-Жюль

Автор:
Валерий Семёнов (Минск)

Вопрос 36:
В эпизоде рассказа Уильяма Шанна разочарованный в жизни герой
присваивает ИМ имена Шелли, Джейн, Раджани, Надин и Эллен и раздумывает,
которая из бывших жен его доконает. В этом эпизоде описан самый жесткий
вариант... Чего?

Ответ:
Русской рулетки.

Комментарий:
Герой дает пулям имена своих бывших жен. Самый жесткий вариант русской
рулетки - это когда в барабане пять пуль.

Источник:
У. Шанн. Чудодейственное средство. http://flibusta.is/b/119615/read

Автор:
Ольга Ярошенко (Могилев)

Тур:
3 этап

Дата:
08-Dec-2017

Редактор:
1-18 - Аркадий Рух; 19-36 - Павел Свердлов

Инфо:
Аркадий Рух благодарит за тестирование Яну Азриэль, Елену Антонову,
Антона Волосатова, Садига Гамидова, Владимира Городецкого, Екатерину
Дубровскую, Александра Зинченко, Ирину Зубкову, Лидию Иоффе, Николая
Коврижных, Дмитрия Когана, Андрея Кокуленко, Наталью Комар, Евгения
Кононенко, Галину Кузьмину, Александра Либера, Данила Маргулиса, Ирину
Марзан, Елену Пивень, Дмитрия Сахарова, Дмитрия Слоуща. Павел Свердлов
благодарит Бога, родителей и киноакадемию.

Вопрос 1:
   <раздатка>
   Мой юный друг, спросивший о странице!
   Неумолимо время, и былых
   Страниц уж нет на тех местах.
   </раздатка>
   Какую односложную фамилию, по мнению Леонида Каганова, должен носить
автор приведенных строк?

Ответ:
Чоч.

Комментарий:
По имени Еггог. Каганов шутит об ошибке 404.

Источник:
https://lleo.me/arhive/no_humor/err2000.shtml

Автор:
Аркадий Рух

Вопрос 2:
В одном из романов Джорджа Мартина присутствует эпизодический персонаж,
который выжил после того, как ему сильным ударом спереди разбили шлем.
Назовите фамилию этого персонажа.

Ответ:
Поттер.

Комментарий:
Такая вот аллюзия на Гарри Поттера, на лбу которого имелся шрам от удара
Волан-де-Морта. Мартин любит вставлять в свои книги такие "пасхалки".

Источник:
https://7kingdoms.ru/wiki/Робин_Поттер

Автор:
Аркадий Рух

Вопрос 3:
Герой песни Семена Слепакова отдыхает, глядя российские криминальные
сериалы, и признаётся, что иногда он "за ментов", иногда "за воров", а
иногда... Закончите одним словом.

Ответ:
Засыпает.

Источник:
https://www.youtube.com/watch?v=tJzx4PTvXis

Автор:
Аркадий Рух

Вопрос 4:
По одной из версий, ОН был изобретен еще в конце 80-х годов XX века
Кэтрин Хеттингер, которая увидела, как палестинские дети бросают камни в
израильских солдат, и решила направить их энергию в мирное русло.
Назовите ЕГО.

Ответ:
Спиннер.

Источник:
https://ru.wikipedia.org/wiki/Спиннер

Автор:
Аркадий Рух

Вопрос 5:
Внимание, в вопросе есть замена.
   Японский персонаж романа Орсона Скотта Карда говорит, что Япония
искупила свои преступления во Второй мировой войне после атомных
бомбардировок Хиросимы и Нагасаки. Его учитель в ответ возмущенно
спрашивает, не стали ли, по его мнению, японцы ИКСАМИ. В современной
Японии количество ИКСОВ составляет около трех процентов. Какое слово мы
заменили ИКСАМИ?

Ответ:
Христиане.

Комментарий:
Воздаяние за грехи - понятие христианской этики, чуждое японской
традиции.

Источник:
О.С. Кард. Дети разума. http://flibusta.is/b/68275/read

Автор:
Аркадий Рух

Вопрос 6:
17 мая 1536 года король Англии Генрих VIII помиловал свою жену, после
чего пришлось вызывать из-за границы представителя ЭТОЙ ПРОФЕССИИ.
Удивительно, но английского специалиста должного уровня найти не
удалось. Назовите ЭТУ ПРОФЕССИЮ.

Ответ:
Палач.

Комментарий:
Королевское помилование состояло в том, что казнь через сожжение на
костре было заменено для Анны Болейн обезглавливанием. При Генрихе казни
в Англии были обычным делом.

Источник:
   1. https://ru.wikipedia.org/wiki/Болейн,_Анна
   2. https://ru.wikipedia.org/wiki/Помилование

Автор:
Аркадий Рух

Вопрос 7:
Статья на сайте antropogenez.ru [антропогенЕз ру], активно занимающемся
борьбой со лженаукой, называется "БаальбЕк: без ГМО". В одном из слов
предыдущего предложения мы заменили две буквы. Восстановите измененное
слово.

Ответ:
НЛО.

Зачет:
UFO; УФО.

Комментарий:
В статье доказывается, что строительство древнего храмового комплекса на
вершине холма Телль Баальбек в Ливане обошлось без вмешательства
пришельцев, как бы "уфологи" ни пытались доказать обратное.

Источник:
http://antropogenez.ru/review/881/

Автор:
Аркадий Рух

Вопрос 8:
В рассказе Василия МидЯнина редактор литературного журнала настойчиво
рекомендует сотрудникам вернуться к работе, так как сам за них трудиться
не собирается. Назовите фамилию этого редактора.

Ответ:
Пушкин.

Комментарий:
"А работать кто будет? Пушкин?".

Источник:
В. Мидянин. Что делать, Фауст? http://flibusta.is/b/276766/read

Автор:
Аркадий Рух

Вопрос 9:
[Ведущему: четко прочитать "все возможные" как два слова.]
   В фантастической повести "Фабрика" описывается таинственный завод,
производящий все возможные цвета. В предыдущем предложении мы пропустили
одну букву. В каком слове?

Ответ:
"Фарбрика".

Комментарий:
Во многих восточноевропейских языках, включая белорусский, слово "фарбы"
переводится как "краски". Логично, что цвета производятся не на Фабрике,
а на Фарбрике.

Источник:
К.А. Терина. Фарбрика. http://flibusta.is/b/515291/read

Автор:
Аркадий Рух

Вопрос 10:
В 2011 году один американский журнал опубликовал статью с описанием
стратегического плана, разработанного военным ведомством США. Статья
называлась "У Пентагона есть план, как остановить наступление ПРОПУСКА".
Впрочем, сами военные заявили, что речь шла лишь о "творческом
упражнении". Заполните пропуск словом, пишущимся через дефис.

Ответ:
Зомби-апокалипсис.

Комментарий:
Вместо привычного "потенциального противника" было предложено
разработать стратегию действий на случай атаки живых мертвецов.

Источник:
https://ru.wikipedia.org/wiki/CONOP_8888

Автор:
Аркадий Рух

Вопрос 11:
Британский сверхтяжелый танк ТОГ-2 разрабатывался с 1941 по 1944 год. По
окончании работ над проектом танк был направлен на военную базу
БОвингтон, где до сих пор находится... Ответьте словом греческого
происхождения: что именно?

Ответ:
Музей.

Комментарий:
К концу войны нужда в этом восьмидесятитонном чудовище отпала, так что
единственный построенный экземпляр был сразу же передан Бовингтонскому
танковому музею.

Источник:
https://warhead.su/2017/08/23/zheleznyy-kaput

Автор:
Аркадий Рух

Вопрос 12:
В пародии на одну популярную телепередачу ведущий предлагает зрителям
слушать танец, нюхать рост, щупать громкость и ДЕЛАТЬ ЭТО. Какие два
слова мы заменили словами "ДЕЛАТЬ ЭТО"?

Ответ:
Смотреть "Голос".

Источник:
https://www.1tv.ru/shows/bolshaya-raznica-tv/parodii/parodiya-na-peredachu-golos-bolshaya-raznitsa-fragment-vypuska-ot-27-01-2013

Автор:
Аркадий Рух

Вопрос 13:
Перед началом показа фильма Роберта Родригеса "Дети шпионов - 4"
посетителям некоторых американских кинотеатров раздавались специальные
карточки, на которых в определенные моменты предлагалось стирать одну из
меток. Какую букву в предыдущем предложении мы пропустили?

Ответ:
D.

Зачет:
Д.

Комментарий:
Фильм имел маркетинговое название "Дети шпионов - 4D". В нужный момент
зрителям предлагалось стирать защитный слой с ароматических ячеек, чтобы
ощутить тот же запах, что и герои фильма.

Источник:
https://ru.wikipedia.org/wiki/Дети_шпионов_4D

Автор:
Аркадий Рух

Вопрос 14:
Внимание, в вопросе есть замена.
   В 1972 году советский инженер-строитель Эммануил Гендель был
направлен в Самарканд, чтобы снести минарет одной из местных мечетей.
Интересно, что знаменитый европейский город воспользоваться услугами
Генделя не пожелал. Какое слово в вопросе мы заменили?

Ответ:
Выпрямить.

Зачет:
Выровнять и т.п. по смыслу.

Комментарий:
Эммануил Гендель - крупнейший советский специалист в области выпрямления
и передвижения зданий. Минарет мечети Биби-Ханым в Самарканде был
"падающей башней", однако Генделю удалось выпрямить его. А вот Пизанская
башня, к радости туристов, так и остается наклонной.

Источник:
https://ru.wikipedia.org/wiki/Гендель,_Эммануил_Матвеевич

Автор:
Аркадий Рух

Вопрос 15:
Николай Рерих назвал свою картину, изображающую летящую по ночному небу
комету, "АЛЬФА" - ведь, по мнению художника, АЛЬФА сияет ярко, но
недолго. В СССР АЛЬФЫ появились 4 ноября 1939 года. Назовите АЛЬФУ двумя
словами точно.

Ответ:
Звезда героя.

Комментарий:
Кометы, как и герои, сияют ярко, но недолго. Золотая звезда Героя
Советского Союза вручалась с 1939 по 1991 год.

Источник:
   1. http://nsk.sibro.ru/photo/poster/detail/2814
   2. https://ru.wikipedia.org/wiki/Медаль_%C2%ABЗолотая_Звезда%C2%BB_(СССР)

Автор:
Аркадий Рух

Вопрос 16:
По словам литературоведа Владимира ЛУкова, "АЛЬФА" Проспера Мериме
получила свое название потому, что построена по принципу АЛЬФЫ: каждая
из составляющих ее новелл "передает какую-то одну характерную черту,
жест, душевное движение", которые складываются в общую картину. Назовите
АЛЬФУ.

Ответ:
Мозаика.

Комментарий:
Сборник новелл "Мозаика", который Мериме выпустил в 1833 году,
действительно состоит из очень разных новелл, никак между собой не
связанных, однако совокупно показывающих всё многообразие жизни.

Источник:
http://19v-euro-lit.niv.ru/19v-euro-lit/lukov-vl-merime/2-2-novelly-1829-1830.htm

Автор:
Аркадий Рух

Вопрос 17:
После того как в Мюнстере открылась выставка, посвященная современному
искусству в общественном пространстве, журналистка Анна ТолстОва назвала
его высококультурным городом. Какие две буквы в предыдущем предложении
мы пропустили?

Ответ:
с, п.

Комментарий:
На выставке были представлены монументальные скульптуры, вписывающиеся в
городской ландшафт, поэтому Мюнстер был назван ВЫСОКОСКУЛЬПТУРНЫМ.

Источник:
https://www.kommersant.ru/doc/3329309

Автор:
Аркадий Рух

Вопрос 18:
Математик Лука ПачОли посвятил свой трактат "О божественной пропорции"
герцогу ЛодовИко СфОрца. Кто выполнил иллюстрации к этому трактату?

Ответ:
Леонардо да Винчи.

Источник:
https://ru.wikipedia.org/wiki/Пачоли,_Лука

Автор:
Аркадий Рух

Вопрос 19:
(pic: 20170884.jpg)
   Изначально рекламным слоганом пародийной комедии "Scary Movie 2"
[скЭри мУви два] планировалось сделать фразу "We lied" [уИ лайд] - "Мы
лгали". Какие два слова мы скрыли от вас на раздатке?

Ответ:
No sequel [чтецу: нОу сИкуэл].

Зачет:
Без продолжения.

Комментарий:
Посмотрите, вам раздали изображение постера первой части фильма. На нем
было написано "No mercy. No shame. No sequel", но потом вышла вторая
часть.
   Мы продолжаем наш синхрон. Второй тур - так сказать, сиквел!

Источник:
http://www.tramvision.ru/lapsus/2005/scary2.shtml

Автор:
Егор Сидорович

Вопрос 20:
В мультфильме "Маша и медведь" на хоккейной клюшке можно увидеть надпись
"Чемпион 1942". Какой символ мы заменили в предыдущем предложении?

Ответ:
2.

Комментарий:
На клюшке написано "Чемпион 1242" - год ледового побоища.

Источник:
Мультсериал "Маша и медведь", 17-я серия "Маша + каша".
https://www.youtube.com/watch?v=AqYsbttfgaQ

Автор:
Алексей Евдоченко

Вопрос 21:
В окончательный вариант фильма "Белое солнце пустыни" не вошла сцена сна
одного из персонажей. В ней он делает то же, что и герой русской песни,
но не один раз, а несколько раз подряд. Назовите этого героя песни.

Ответ:
Степан Разин.

Комментарий:
Красноармейцу Сухову снится, что он - Степан Разин. Бросать в набежавшую
волну приходится не одну восточную женщину, а нескольких.

Источник:
https://sadalskij.livejournal.com/3262610.html

Автор:
Егор Сидорович

Вопрос 22:
   <раздатка>
   [пропуск1] are from Clash of Clans, [пропуск2] are from Candy Crush
   </раздатка>
   Перед вами заголовок статьи об аудиториях мобильных игр, в котором мы
пропустили два слова. Заполните каждый из пропусков одним словом.

Ответ:
Men, Women.

Зачет:
Мужчины, женщины.

Комментарий:
"Men are from Clash of Clans, women are from Candy Crush". Обыгрывается
заголовок книги Джона Грэя "Men Are from Mars, Women Are from Venus" -
"Мужчины с Марса, женщины с Венеры".

Источник:
   1. https://news.usc.edu/115999/men-are-from-clash-of-clans-women-are-from-candy-crush
   2. https://en.wikipedia.org/wiki/Men_Are_from_Mars,_Women_Are_from_Venus

Автор:
Александра Бурчалова

Вопрос 23:
Автор зомби-хоррора "Некрополитен" парадоксальным образом замечает, что
"зомби появились в метрополитене внезапно". Какие три слова мы заменили
в предыдущем предложении одним?

Ответ:
Как из-под земли.

Зачет:
Словно из-под земли.

Источник:
https://rudy-de.livejournal.com/255380.html

Автор:
Павел Свердлов

Вопрос 24:
Статья на сайте "Российской газеты" о новой услуге в московском
метрополитене называется "Из-под земли достанут". Какие две буквы мы
заменили в предыдущем предложении?

Ответ:
вя.

Комментарий:
"Доставят". В метро будут размещаться сервисы по доставке почтовых
отправлений.

Источник:
https://rg.ru/2016/08/04/reg-cfo/v-stolichnom-metro-nachnut-otpravliat-pisma-i-mini-posylki.html

Автор:
Ольга Потапова

Вопрос 25:
Газета "The Telegraph" в статье о Марин Ле Пен писала, что в случае
победы французский политик обещала УЙТИ ПО-АНГЛИЙСКИ. Какой неологизм мы
заменили в предыдущем предложении?

Ответ:
Фрекзит.

Зачет:
По созвучию.

Комментарий:
Весной 2017 года во Франции проходили выборы президента, и кандидат
Марин Ле Пен в случае своей победы обещала французам выход из ЕС,
который уже потенциально окрестили Фрекзитом.

Источник:
https://www.telegraph.co.uk/news/worldnews/europe/france/11696466/Call-me-Madame-Frexit-Front-National-leader-Marine-le-Pen-says.html

Автор:
Алексей Евдоченко

Вопрос 26:
В книге "Датская модель" говорится, что высокие налоги и преобладание
бюджетного сектора обычно подавляют экономический рост, инновации и
конкурентоспособность. Экономическая наука считает такую модель
нежизнеспособной. Далее датская экономика называется экономикой ЕГО.
Кого?

Ответ:
Шмеля.

Комментарий:
По аналогии со шмелем, который по законам физики летать не может,
датская экономическая модель в теории обречена на провал.

Источник:
М. Бут. Почти идеальные люди. Вся правда о жизни в "Скандинавском раю".
http://flibusta.is/b/494052/read

Автор:
Алексей Евдоченко

Вопрос 27:
   <раздатка>
   "Hm! Hm! Hm! Hm!"
   </раздатка>
   Перед вами название фрагмента известной оперы, который исполняет
только что наказанный персонаж. Суть наказания состоит в воплощении в
жизнь известной идиомы в буквальном смысле. Назовите идиому.

Ответ:
[Закрыть] рот на замок.

Зачет:
[Держать] рот на замке; прочие ответы, которые являются идиомами и
означают закрытый насильственно рот.

Комментарий:
Папагено, персонаж "Волшебной флейты", наказан за вранье и хвастовство
надеванием на рот замка. В таком виде он пытается пожаловаться принцу
Тамино на свое бедственное положение.

Источник:
Опера Моцарта "Волшебная флейта", диалог Папагено и принца Тамино.

Автор:
Ольга Потапова

Вопрос 28:
Во времена президентства Урхо Кекконена отношения Финляндии и СССР
значительно улучшились. За близость к социалистическому лагерю на Западе
придумали для Финляндии прозвище, созвучное с названием другой, ныне не
существующей страны. Напишите это прозвище.

Ответ:
Кеккословакия.

Источник:
Ben Livson. The Life of Dr. Mikael Livson.
https://books.google.ru/books?id=jM2jg-AU-P8C&pg=PA31#v=onepage&q&f=false

Автор:
Алексей Евдоченко

Вопрос 29:
Герои 238-й серии мультсериала "Лунтик" испытывают отчаяние и страх
нападения, пока на помощь им не приходят светлячки. Название серии
совпадает с названием фильма 1980 года, снятого по одноименной книге.
Как называется этот фильм?

Ответ:
"Сияние".

Источник:
   1. Мультсериал "Лунтик", 238-я серия "Сияние".
https://www.youtube.com/watch?v=zMvkwbqoRc0
   2. https://www.kinopoisk.ru/film/siyanie-1980-409/

Автор:
Яна Самойлович

Вопрос 30:
(pic: 20170885.jpg)
   Какой псевдоним из одного слова взял серийный убийца, в письмах
которого содержались такие знаки?

Ответ:
Зодиак.

Комментарий:
Мы раздали вам 12 знаков. Псевдоним напрашивается сам собой.

Источник:
   1. http://retrobazar.com/journal/izvestnye-ljudi/1004_zodiak-istorija-serijnogo-ubijcy.html
   2. https://thoughtcatalog.com/julia-metraux/2018/09/7-insane-but-weirdly-believable-theories-about-the-zodiac-killer/

Автор:
Павел Свердлов

Вопрос 31:
Считается, что пасмурная погода в Сиэтле является одной из причин
повышенного уровня самоубийств. Поэтому неудивительно, что штат
Вашингтон стал одним из первых, где СДЕЛАЛИ ЭТО. Какие два слова,
начинающиеся на соседние буквы алфавита, мы заменили словами "СДЕЛАЛИ
ЭТО"?

Ответ:
Легализовали марихуану.

Комментарий:
Вот такой способ борьбы с самоубийствами.

Источник:
   1. https://lookatusa.com/geo/goroda/seattle/al-stl.html
   2. https://www.kommersant.ru/doc/3150093

Автор:
Алексей Евдоченко

Вопрос 32:
Герой сериала "Безумцы", действие которого происходит в шестидесятых,
починил одно устройство и продемонстрировал, что оно работает. За это
его назвали быстрым коричневым лисом. Какое устройство он починил?

Ответ:
Пишущая машинка.

Зачет:
Печатная машинка.

Комментарий:
"The quick brown fox jumps over the lazy dog" - англоязычная панграмма.
Панграммами обычно тестируют работу печатной машинки. Действие сериала
"Безумцы" происходит в 1960-х годах, этим можно отсечь клавиатуру.

Источник:
   1. Телесериал "Безумцы", s07e13.
   2. https://en.wikipedia.org/wiki/The_quick_brown_fox_jumps_over_the_lazy_dog

Автор:
Александра Бурчалова

Вопрос 33:
Один блогер сетует, что лопапейсу, продающуюся в сувенирных магазинах
Исландии, ВАЛЯТ на ОТВАЛИ. Напишите любое из слов, которые мы заменили в
предыдущем предложении.

Ответ:
Вяжут.

Зачет:
Отвяжись.

Комментарий:
Кстати, лопапейса - это такой свитер. Это ведь нормально, что в Исландии
в сувенирных магазинах продаются свитера?

Источник:
https://tchainka.livejournal.com/507075.html

Автор:
Ольга Потапова

Вопрос 34:
Герой рассказа Дональда Уэстлейка называет полицейские рации слуховым
эквивалентом ЕГО: из небольшой коробочки доносятся только треск и
повизгивание, но полицейские всё понимают. Назовите ЕГО двумя словами.

Ответ:
Врачебный почерк.

Зачет:
Почерк врачей, медицинский рецепт и т.п. по смыслу.

Комментарий:
Врачи тоже как-то понимают всё, написанное другими врачами.

Источник:
Д. Уэстлейк. Воровская дюжина. http://flibusta.is/b/514231/read

Автор:
Егор Сидорович

Вопрос 35:
Мэт Ширли описывал ИКС в виде диаграммы с восходящими кривыми чувства
паники и времени, потраченного на просмотр сериалов, а также нисходящей
кривой желания жить. В сообществе wise_advice на портале
www.liveinternet.ru ИКС образно сравнивают с концом августа. Назовите
ИКС двумя словами, начинающимися на одну и ту же букву.

Ответ:
Вечер воскресенья.

Источник:
   1. http://medialeaks.ru/2310qaz-diagrammyi-na-kazhdyiy-den
   2. http://www.liveinternet.ru/community/wise_advice/post397220273/
   3. http://www.cluber.com.ua/lifestyle/poleznyie-sovetyi/2017/03/vot-chto-uspeshnyie-lyudi-delayut-v-voskresene-vecherom/

Автор:
Станислав Габрусевич

Вопрос 36:
На выставке в Музее современного искусства в Нью-Йорке представлено 111
моделей одежды и аксессуаров, оказавших наибольшее влияние на моду
XX-XXI веков. Назовите фирму-производителя предмета, находящегося первым
в списке экспонатов.

Ответ:
"Levi's".

Комментарий:
Список был отсортирован по порядку 0-9, A-Z. Первым предметом,
представленным на выставке, была модель джинсов 501 Jeans by Levi's.

Источник:
   1. https://static.dezeen.com/uploads/2017/09/items-is-fashion-modern-moma_dezeen_2364_col_1-852x569.jpg
   2. https://medium.com/items/items-is-fashion-modern-checklist-e353b83e7652
   3. http://www.levi.com/US/en_US/features/501-series (название модели
на сайте производителя)

Автор:
Ольга Потапова

Тур:
4 этап

Дата:
16-Feb-2018

Редактор:
Иван Топчий и Павел Малецкий (Минск)

Инфо:
Редакторы благодарят за тестирование и ценные замечания: команду "Карты,
простынь, два носка", Валерия Семёнова, Александра Курзинера, Валентина
Копочеля, Ивана Мозолюка, Алексея Волчка, Алексея Гончарова, Гражину
Бобилевич, Германа Чепикова, Арину Арзамазову, Екатерину Мудрагель.

Вопрос 1:
[Нулевой вопрос]
   Внимание, в вопросе есть замена.
   Статья на тематическом сайте рассказывает, что в некоторых монастырях
можно попробовать и миньон - но его монахи готовят обычно для себя,
чтобы исполнять свои обязанности после трапезы надлежащим образом. Какое
слово мы заменили в этом вопросе?

Ответ:
Сингл.

Комментарий:
Речь в статье идет не о стейках, а о пиве. В частности, о популярных
бельгийских аббатских сортах, таких как дуббель, трипель и квадрупель.
Для собственных нужд монахи варят светлое пиво с низким содержанием
алкоголя, чтобы находиться в трезвом уме при исполнении своих служебных
обязанностей. Сингл и миньон - это еще и типы музыкальных пластинок.

Источник:
   1. https://www.pivo.by/articles/reviews/belgian-beer
   2. https://ru.wikipedia.org/wiki/Сингл
   3. https://ru.wikipedia.org/wiki/Мини-альбом

Автор:
Алексей Гончаров (Минск)

Вопрос 2:
В начале XIX века в США появилась настольная игра "Особняк счастья".
Игра учила детей христианским ценностям, поэтому привычного предмета в
ней не было, и его роль исполнял ИКС. Такая роль ИКСА нам с вами не
должна показаться необычной. Назовите слово, которое мы заменили ИКСОМ.

Ответ:
Волчок.

Комментарий:
[Ведущему: желательно объявить автора вопроса.]
   Обычно в играх для определения случайного числа используется
игральный кубик, но он четко ассоциируется с азартными играми. В
"Особняке счастья" использовался особый волчок с числами на гранях. В
такой же роли - выбор случайного вопроса - используется волчок в
элитарном клубе "Что? Где? Когда?".

Источник:
https://en.wikipedia.org/wiki/The_Mansion_of_Happiness

Автор:
Алексей Волчок (Минск)

Вопрос 3:
Последний оргАн фирмы "ВУрлитцер" был произведен в начале 1940-х. Он мог
воспроизводить звуки ветра, грома, клаксонов и многие другие. Мы не
спрашиваем, какие четыре буквы пропущены в тексте вопроса. Назовите
учреждения, в которые поставлялись эти инструменты.

Ответ:
Кинотеатры.

Зачет:
Кинозалы.

Комментарий:
Специальные киноорганы могли не только имитировать пианино, но и
воспроизводить другие звуки для немого кино. Из-за распространения
звукового кино их перестали производить в начале 1940-х.

Источник:
https://ru.wikipedia.org/wiki/Театральный_орган

Автор:
Алексей Волчок (Минск)

Вопрос 4:
Блиц.
   Каждый стикер из набора "Философия" в Telegram [телегрАм] состоит из
портрета знаменитого человека и подписи, которую тот якобы произносит.
   1. Назовите человека, изображенного на стикере с надписью "Не думаю".
   2. Назовите человека, изображенного на стикере с надписью "Ничего не
знаю".
   3. Назовите человека, изображенного на стикере с надписью "Сказал как
отрезал".

Ответ:
   1. [Рене] Декарт.
   2. Сократ.
   3. [Уильям] Оккам.

Комментарий:
В первом вопросе отсылка к словам Декарта "Мыслю, следовательно
существую", во втором - к словам Сократа "Я знаю, что ничего не знаю", в
третьем речь идет о принципе "Бритва Оккама".

Источник:
Набор стикеров "Философия" в Telegram.
https://tlgrm.ru/stickers/Philosophy_Guys

Автор:
Антон Шевченя (Минск)

Вопрос 5:
Рекламный слоган Беларусбанка предлагает сделать незабываемым ЕГО.
Назовите ЕГО словом английского происхождения.

Ответ:
Пин-код.

Комментарий:
Беларусбанк предлагает клиентам самим придумать себе пин-код, чтобы уже
точно не забыть его. Пин-код обычно состоит из четырех цифр, вопрос в
туре стоит четвертым, что тоже могло помочь некоторым командам. А могло
и не помочь.

Источник:
https://www.belarusbank.by/ru/33139/press/bank_news/31842

Автор:
Павел Малецкий (Минск)

Вопрос 6:
Андреас Кацулас исполнял роль инопланетного посла Г'Кара в сериале
"Вавилон-5". Когда его спросили, почему он называет своего персонажа
ЖэкАром, тот ответил: "Я решил, что он - ПРОПУСК". В итоге это хорошо
сочеталось с вычурным непрактичным шарфом, который носил персонаж.
Назовите слово, которое мы пропустили.

Ответ:
Француз.

Источник:
"Вавилон-5. Начало", комментарии создателей.

Автор:
Алексей Волчок (Минск)

Вопрос 7:
[Ведущему: очень четко прочитать окончание в слове "спасениЯ" и слегка
выделить интонационно слово "миллиардов".]
   По подсчетам пользователя портала "Quora" [квуОра], на спасения
персонажей этого актера ушло не менее 900 миллиардов долларов. Назовите
этого актера.

Ответ:
[Мэтт] Деймон.

Комментарий:
Мэтт Деймон - известный американский актер, персонажей которого в
множестве фильмов пытаются спасти. Например, в фильмах "Марсианин",
"Спасти рядового Райана" и "Интерстеллар".

Источник:
https://www.quora.com/How-much-money-has-been-spent-attempting-to-bring-Matt-Damon-back-from-distant-places

Автор:
Иван Мозолюк (Минск)

Вопрос 8:
Однажды в гости к ЕГО родителям пришел Иван Дмитриев и решил подшутить
над ребенком, сказав: "Какой арабчик!". Но десятилетний мальчик не
растерялся и ответил: "Зато не рябчик!", намекая на рябое от оспы лицо
Дмитриева. Кто был этим находчивым мальчиком?

Ответ:
[Александр Сергеевич] Пушкин.

Комментарий:
Ну а кто ж еще? Редакторы намеренно поставили этот вопрос седьмым. Ведь,
если учитывать нулевой вопрос, он как раз получается восьмым.

Источник:
https://kulturologia.ru/blogs/060618/39225/

Автор:
Ксения Драгунова (Минск)

Вопрос 9:
   <раздатка>
   Blankadas velo unusola
   En la nebula mara blu',
   &#284;i kion lasis, kion volas
   en fremdaj landoj ser&#265;i plu?
   </раздатка>
   Историк Линкольн Пейн пишет, что ИКС в полную силу развернулся в
XVIII веке. Назовите слово, которое мы заменили ИКСОМ.

Ответ:
Парус.

Комментарий:
В раздатке - начало стихотворения Лермонтова "Парус" в переводе на
эсперанто. Слово "развернулся" - небольшая подсказка.

Источник:
   1. Л. Пейн. Море и цивилизация. Мировая история в свете развития
мореходства.
https://books.google.ru/books?id=7tcyDwAAQBAJ&pg=PT828#v=onepage&q&f=false
   2. https://eo.wikipedia.org/wiki/Velo_(versa%C4%B5o)

Автор:
Константин Егиазаров, Валерий Семёнов (Минск)

Вопрос 10:
В юмористическом фэнтезийном романе героям объясняют происхождение
местной достопримечательности. По легенде, один великан оскорбил богов,
и те погрузили его в землю так, что над поверхностью осталась только
одна часть тела. Ответьте точно: какая именно?

Ответ:
Средний палец [руки].

Комментарий:
Без комментариев.

Источник:
Ю.А. Фирсанова. Наколдованная любовь.
https://books.google.ru/books?id=YlvcDAAAQBAJ&pg=PT91#v=onepage&q&f=false

Автор:
Ольга Берёзко (Минск)

Вопрос 11:
Один из разделов статьи в журнале "Популярная механика" называется
"Хлопок одной ладонью". В нем упоминаются Майкл Фарадей, Джеймс Максвелл
и, конечно, Дирак. Назовите объект этой статьи двумя словами, которые
начинаются на одну и ту же букву.

Ответ:
Магнитный монополь.

Комментарий:
Майкл Фарадей и Джеймс Максвелл были основоположниками
электромагнетизма. Поль Дирак (имя ученого - небольшая подсказка)
предположил существование частицы с магнитным зарядом, которая, правда,
до сих пор не обнаружена. Магнитный монополь в статье образно
сравнивается с хлопком одной ладонью - одним из центральных понятий
дзен.

Источник:
   1. https://www.popmech.ru/science/6791-dzhinn-iz-lukovitsy-magnit-ob-odnom-polyuse/
   2. https://ru.wikipedia.org/wiki/Магнитный_монополь

Автор:
Алексей Гончаров (Минск)

Вопрос 12:
В мае 2011 года Трисс Меригольд - привлекательная чародейка, героиня
саги о Ведьмаке - появилась в НЕМ и даже "дала" интервью. Назовите ЕГО
двумя словами, которые начинаются на одну и ту же букву.

Ответ:
Польский "Плэйбой".

Источник:
http://www.kaermorhen.ru/modules.php?name=News&file=view&sid=1234

Автор:
Ксения Драгунова (Минск)

Вопрос 13:
Ричард Уильямс едва не потерял обе ноги из-за обморожения, проведя
несколько часов в ледяной воде. Однако через 12 лет в Париже он смог
стать олимпийским чемпионом по теннису. К сожалению, его отец погиб и не
разделил с ним радость победы. А где отец Уильямса провел свой последний
день?

Ответ:
На "Титанике".

Зачет:
По слову "Титаник".

Комментарий:
Уильямс и его отец были пассажирами "Титаника". При крушении Ричард
доплыл до перевернутой шлюпки и просидел на ней по колено в воде, из-за
чего и отморозил ноги. А вот отец Уильямса в крушении не выжил. Кстати,
уже в 1912 году Уильямс выиграл US Open.

Источник:
   1. http://sport.tut.by/news/aboutsport/444590.html
   2. https://en.wikipedia.org/wiki/R._Norris_Williams
   3. https://www.tennisfame.com/hall-of-famers/richard-williams

Автор:
Александр Курзинер (Минск)

Вопрос 14:
Изначально британский писатель планировал назвать роман "Мертвый, не
мертвый". Однако издатель сначала сократил заглавие до "Не мертвый", а в
последний момент и вовсе заменил его именем собственным. Каким?

Ответ:
Дракула.

Комментарий:
Заглавный герой действительно не то чтобы мертв, но и не то чтобы жив.
   Для тех, кто написал в ответной карточке слово "Франкенштейн",
сообщаем, что название романа Мэри Шелли целиком звучит как
"Франкенштейн, или Современный Прометей". Да и написан он не писателем,
а писательницей.

Источник:
https://www.adme.ru/tvorchestvo-pisateli/14-legendarnyh-knig-kotorye-avtory-sobiralis-nazvat-sovsem-po-drugomu-1412415/

Автор:
Мария Орановская-Малецкая (Минск)

Вопрос 15:
В вопросе есть замена.
   Политическую ситуацию в России перед развалом империи Петр Столыпин
описал словом "АЛЬФА". Продолжение игры "АЛЬФА" получило название "Осада
Белого дома". Назовите АЛЬФУ.

Ответ:
Перестройка.

Зачет:
Perestroika.

Комментарий:
"Не то, конечно, в стране, находящейся в периоде перестройки, а
следовательно, и брожения", - говорил Столыпин. Восьмьюдесятью годами
спустя новая Перестройка тоже запустила процессы, приведшие к развалу
империи. Осада Белого дома в Москве происходила во время
конституционного кризиса в октябре 1993 года.

Источник:
   1. М.В. Зыгарь. Империя должна умереть.
http://flibusta.is/b/521912/read
   2. https://ru.wikipedia.org/wiki/Perestroika

Автор:
Валентин Копочель (Минск)

Вопрос 16:
9 мая 2017 года ветеран баскетбола Ману Джинобили поставил решающий
блок-шот на последних секундах матча плей-офф НБА. Итог этой встречи
один из русскоязычных фанатов прокомментировал известной фразой из
четырех слов. Воспроизведите ее.

Ответ:
Спасибо деду за победу.

Комментарий:
Блок-шот Джинобили позволил команде "Сан-Антонио" выиграть важную
встречу. Иронизируя над возрастом 39-летнего на момент матча
баскетболиста, один из болельщиков написал "Спасибо деду за победу".
Дата матча - 9 мая и слово "ветеран" были небольшими подсказками.

Источник:
https://www.sports.ru/basketball/1051104804.html

Автор:
Валентин Копочель (Минск)

Вопрос 17:
[Ведущему: сделать небольшую паузу между "Ай" и "Тен".]
   Среди американских болельщиков встречи баскетбольных команд "Хьюстон
Рокетс" и "Сан-Антонио Спёрз" получили прозвище Ай-Тен. Дело в том, что
Ай-Тен - это... Что?

Ответ:
Трасса, соединяющая Хьюстон и Сан-Антонио.

Зачет:
По словам "трасса", "дорога" или "шоссе" с упоминанием городов или без
него.

Источник:
https://ru.wikipedia.org/wiki/I-10

Автор:
Павел Малецкий (Минск)

Вопрос 18:
Автор вышедшей в самом конце 2017 года статьи "Легенда ПРОПУСК"
рассуждает о том, что удачный фильм просто обречен на сиквелы, ремейки и
подражания. Какие три символа мы пропустили?

Ответ:
N 18.

Комментарий:
Статья посвящена только что вышедшему в прокат фильму "Движение вверх".
Ее автор рассуждает о сюжетной и стилистической схожести новинки и
фильма "Легенда N 17". Поэтому и называет "Движение вверх" легендой N
18. Данный вопрос - семнадцатый в туре. Но с учетом нулевого он и 17-й,
и 18-й одновременно.

Источник:
https://www.kp.by/daily/26776/3809458/

Автор:
Павел Малецкий (Минск)

Вопрос 19:
В период НЭПа в Советской России наблюдались послабления по многим
аспектам жизни общества. Так, 1 мая 1921 года многие петроградцы
предпочли ИКС большевистской демонстрации. Где проходит ИКС на картине
второй половины XIX века?

Ответ:
В Курской губернии.

Комментарий:
В 1921 году 1 мая совпало с днем Пасхи. В период НЭПа религия не была
под официальным запретом, поэтому многие петроградцы предпочли
отправиться на крестный ход, а не на демонстрацию. "Крестный ход в
Курской губернии" - известная картина Ильи Репина.

Источник:
   1. Н.Б. Лебина. Советская повседневность: нормы и аномалии. От
военного коммунизма к большому стилю. http://flibusta.is/b/415924/read
   2. https://ru.wikipedia.org/wiki/Крестный_ход_в_Курской_губернии

Автор:
Валерий Семёнов, Павел Малецкий (Минск)

Вопрос 20:
Дуплет.
   1. В начале XX века в Англии купить ИКС можно было за три сотни
фунтов - заработок фермера за шесть лет. Назовите ИКС одним словом.
   2. Русскому поэту-славянофилу Степану Шевырёву принадлежит авторство
фразы "загнивающий Запад". В 1860 году Шевырёв навсегда СДЕЛАЛ ЭТО.
Сделал что?

Ответ:
   1. Трактор.
   2. Уехал из России.

Зачет:
   2. По смыслу.

Источник:
   1. Документальный сериал "Эдвардианская ферма", 3-я серия.
   2. https://ru.wikipedia.org/wiki/Гнилой_Запад
   3. https://ru.wikipedia.org/wiki/Шевырёв,_Степан_Петрович

Автор:
Павел Митар (Минск)

Вопрос 21:
   <раздатка>
   Буржуазные ценности
   </раздатка>
   В названии магазина антиквариата мы пропустили один символ.
Восстановите это название в исходном виде.

Ответ:
Б/уржуазные ценности.

Комментарий:
Антиквариат скорее всего кому-то раньше принадлежал, а значит, формально
б/у.

Источник:
https://www.instagram.com/by_tsennosti/

Автор:
Надежда Лейчинская (Минск)

Вопрос 22:
Сайт для создания реверсов музыкальных файлов называется "15:00"
[пятнадцать ноль ноль]. Что мы заменили в предыдущем предложении?

Ответ:
3pm.

Зачет:
3 PM в любом написании.

Комментарий:
Реверсом называется проигрывание произведения в обратном направлении.
Сайт позволяет пользователю загрузить файлы форматов .ogg, .wav и,
конечно, mp3. 3pm - это mp3 наоборот.

Источник:
https://www.mp3-reverser.com/en/

Автор:
Алексей Гончаров (Минск)

Вопрос 23:
Сторонники БРИТАНСКОГО стиля жизни предпочитают шерстяным свитерам
пальто и не боятся выходить из дома в плохую погоду и спорить со своими
гостями. Недавно появившееся название этого БРИТАНСКОГО стиля в русском
языке совпадает с названием европейского города. Какого?

Ответ:
Брюгге.

Комментарий:
Датское слово "хюгге" означает состояние блаженства и счастья, которое
мы испытываем в компании близких, почувствовав аромат "бабушкиного"
варенья, укутавшись в теплое одеяло и попивая кофе, когда за окном
слякоть и холод. Хюгге стало одним из слов 2017 года по версии словаря
"Collins". Многим британцам пришелся не по вкусу этот тренд, и они
начали противопоставлять свой образ жизни новомодному хюгге. Такой стиль
получил название "брюгге".

Источник:
https://lifehacker.ru/tyomnaya-storona-xygge/

Автор:
Валентин Копочель (Минск)

Вопрос 24:
Потоковый сервис Netflix [нЕтфликс] решил расширить свою аудиторию,
добавив специальную озвучку с подробным описанием происходящего на
экране, включая движения героев, выражения их лиц, интерьер, смену сцен
и многое другое. Назовите персонажа комиксов Марвел, на сериале о
котором было решено опробовать эту технологию.

Ответ:
Сорвиголова.

Зачет:
Daredevil.

Комментарий:
Данная озвучка позволит "смотреть" сериал людям с проблемами зрения, в
т.ч. слепым. "Сорвиголова" ("Daredevil") - сериал об одноименном
супергерое, альтер-эго слепого адвоката Мэта Мёрдока.

Источник:
http://geekcity.ru/sorvigolova-vyjdet-v-formate-dlya-slepyx/

Автор:
Николай Будник (Минск)

Вопрос 25:
[Ведущему: кавычки не озвучивать.]
   В этом вопросе ИКС заменяет слово, которое пишется через дефис.
   В 2007 году в Великобритании был проведен опрос, по итогам которого
"самыми знаменитыми ИКСАМИ в истории человечества" были названы эпизоды,
связанные с Шэрон Стоун и с уроженцем Баку. Что мы заменили ИКСОМ?

Ответ:
Стоп-кадр.

Комментарий:
Самыми знаменитыми стоп-кадрами британцы признали момент, когда Шэрон
Стоун меняет местами скрещённые ноги в сцене допроса из фильма "Основной
инстинкт", а также знаменитый гол Джеффри Хёрста в ворота сборной ФРГ,
который был засчитан по указанию советского линейного арбитра Тофика
Бахрамова и принес англичанам их единственное в истории чемпионство.

Источник:
   1. https://www.eurosport.ru/football/world-cup/2010/story_sto2304132.shtml
   2. https://ru.wikipedia.org/wiki/Бахрамов,_Тофик_Бахрам_оглы

Автор:
Алексей Гончаров (Минск)

Вопрос 26:
[Ведущему: кавычки не озвучивать; четко прочитать слово "отстаивать".]
   Еще несколько десятилетий назад киномеханику на "Востоке" приходилось
отстаивать фильмы, которые планировались к показу. В одно из слов
предыдущего предложения мы добавили одну букву. Восстановите его в
исходном виде.

Ответ:
Оттаивать.

Комментарий:
"Восток" - полярная станция. Из-за низких температур фильмы на пленке
подмерзали, и перед просмотром их нужно было "согреть".

Источник:
В.М. Песков. Зимовка. http://flibusta.is/b/77020/read

Автор:
Надежда Лейчинская (Минск)

Вопрос 27:
   <раздатка>
   Чосон (название Кореи во время правления династии Чосон) в период с
1592 по 1598 годы страдал от набегов японцев (смотрите статью __________
_____ (Корея)).
   </раздатка>
   Какие два слова мы пропустили в розданной вам цитате из Википедии?

Ответ:
Семилетняя война.

Комментарий:
"Семилетняя война (Корея)" - именно так статья называется в Википедии.
Более известная война между европейскими державами обозначена просто как
Семилетняя война.

Источник:
https://ru.wikipedia.org/wiki/История_Кореи#Чосон

Автор:
Алексей Волчок (Минск)

Вопрос 28:
В сражении под Кульмом 10-тысячный русский отряд сдержал 30-тысячный
корпус французов и спас от плена короля Пруссии. После битвы ИХ не
хватило, поэтому солдаты делали ИХ сами, используя металл французских
кирас. Назовите ИХ двумя словами точно.

Ответ:
Железные кресты.

Зачет:
Кульмские кресты.

Комментарий:
Прусский король решил наградить всех участников битвы орденами. Первые
кресты изготавливались из металла трофейных кирас самими награжденными,
после того как они об этом узнали.

Источник:
   1. https://ru.wikipedia.org/wiki/Сражение_под_Кульмом
   2. https://ru.wikipedia.org/wiki/Кульмский_крест

Автор:
Павел Митар (Минск)

Вопрос 29:
В вопросе словом "ИКС" заменены два слова.
   После поражения под Кульмом пленный французский генерал показал
Александру I ИКС, и тот обещал облегчить его участь. В итоге генерал
получил свободу меньше чем через год. Изобразите на бланке любой ИКС.

Ответ:
Глаз в треугольнике.

Зачет:
Циркуль; угольник; циркуль и угольник с буквой G.

Комментарий:
По словам очевидца, генерал показал царю масонский знак, что значительно
улучшило его положение.

Источник:
   1. https://ru.wikipedia.org/wiki/Сражение_под_Кульмом
   2. https://ru.wikipedia.org/wiki/Масонство

Автор:
Павел Митар (Минск)

Вопрос 30:
В 802 году некий Исаак прибыл из Генуи в Ахен ко двору Карла Великого.
При нем были богатые дары от Харун ар-Рашида, в том числе один необычный
подарок. Это позволяет сравнить Исаака с более известным человеком,
жившим примерно на тысячу лет раньше. С кем?

Ответ:
Ганнибал.

Комментарий:
Этим необычным подарком был слон.

Источник:
   1. https://republic.ru/posts/80139
   2. https://ru.wikipedia.org/wiki/Ганнибал

Автор:
Надежда Лейчинская (Минск)

Вопрос 31:
В одном из эпизодов российского комедийного сериала двое друзей
разрешают спор традиционным способом. После этого проигравший называет
победителя ИКСОМ. Некоторое время ИКСОМ был Тирион Ланнистер. Напишите
три слова, которые мы заменили ИКСОМ.

Ответ:
Мастер над монетой.

Комментарий:
Друзья улаживают спор подбрасыванием монетки. Проигравший называет
победителя мастером над монетой. Мастер над монетой - своеобразный
аналог министра финансов Семи королевств из цикла произведений Джорджа
Мартина.

Источник:
???

Автор:
Павел Малецкий (Минск)

Вопрос 32:
Рожденный в Калуге в начале XVII века Иван Дмитриевич в русской
историографии известен как Воронёнок. Напишите в исходном виде слово,
которое мы слегка удлинили.

Ответ:
Ворёнок.

Комментарий:
Сын Марины Мнишек и Лжедмитрия II, прозванного Тушинским Вором.

Источник:
https://ru.wikipedia.org/wiki/Иван_Ворёнок

Автор:
Павел Митар (Минск)

Вопрос 33:
Группа сортов лилейников, первый из которых зарегистрирован в 1933 году,
отличается экзотической и необычной формой цветка. Какой трехбуквенной
аббревиатурой эта группа обозначается в международной классификации?

Ответ:
UFO.

Комментарий:
Сокращение от "Unusual Form" [анъЮжуал форм]. Примерно к тем же годам
относятся первые сообщения о летающих тарелках.

Источник:
   1. http://www.lileynic.com.ua/index.php?id=3
   2. https://en.wikipedia.org/wiki/Unidentified_flying_object

Автор:
Ольга Берёзко (Минск)

Вопрос 34:
Персонаж сериала о зомби-апокалипсисе с удивлением узнаёт, что
преследовавший его охотник за головами сменил имя и присоединился к
группе героев, спасающих человечество. Говоря об этом, персонаж
упоминает дорогу в Дамаск и сравнивает бывшего противника... С кем?

Ответ:
[Апостол] Павел.

Зачет:
Савл.

Комментарий:
Обращение Савла на пути в Дамаск - один из эпизодов жития апостола
Павла, бывшего ранее среди гонителей христиан, который сыграл
колоссальную роль в его обращении в христианство.

Источник:
Телесериал "Nation Z", s03e01.

Автор:
Павел Митар (Минск)

Вопрос 35:
Сейчас правозащитница и адвокат Амаль Аламуддин является женой Джоржда
Клуни, но о ее личной жизни в прошлом не известно ничего. Журналисты
шутили, что, вероятно, всё это время она была занята тем, что прошибала
ИХ. Назовите ИХ двумя словами.

Ответ:
Стеклянные потолки.

Комментарий:
Термин, означающий невидимый и формально никак не обозначенный барьер
("потолок" в карьере), ограничивающий продвижение женщин по служебной
лестнице по причинам, не связанным с их профессиональными качествами.

Источник:
https://meduza.io/feature/2017/06/08/u-dzhordzha-kluni-i-amal-alamuddin-rodilis-bliznetsy-za-chto-my-lyubim-etu-semyu

Автор:
Надежда Лейчинская (Минск)

Вопрос 36:
В передаче о периоде правления Александра II описан такой случай:
"Оброненную рукопись подобрал мелкий чиновник. <...> Здесь, на этой
мостовой, решалось ПРОПУСК с романом ПРОПУСК". Конец цитаты. Какие два
слова мы дважды пропустили в этом вопросе?

Ответ:
Что делать.

Комментарий:
Некрасов обронил рукопись Чернышевского.

Источник:
Проект Леонида Парфёнова "Российская империя". Выпуск 12 "Александр II,
часть 2" (после 33-й минуты).

Автор:
Алексей Волчок (Минск)

Вопрос 37:
При рождении ОН получил имя Джеймс Кливленд. Чтобы прокормить семью, ОН
неоднократно соглашался выступать против лошадей и охотничьих собак.
Назовите ЕГО фамилию.

Ответ:
Оуэнс.

Комментарий:
Более известное имя "Джесси" знаменитый легкоатлет получил,
представившись новой учительнице домашним прозвищем "J.C." (от James
Cleveland). Несмотря на победу на Олимпиаде 1936 года, Оуэнс имел
большие финансовые проблемы в течение всей жизни.

Источник:
   1. http://www.gazettco.com/dzhessi-ouens-legendy-i-fakty/
   2. https://ru.wikipedia.org/wiki/Оуэнс,_Джесси

Автор:
Иван Мозолюк, Иван Топчий (Минск)

Тур:
5 этап

Дата:
16-Mar-2018

Редактор:
Виталий Захарик, Сергей Дубелевич, Сергей Апанович

Вопрос 1:
Итак, мы начинаем наш путь по вопросной дистанции!
   В интерпретации литератора АльгЕрда БахарЕвича жители столицы
принимают ЕГО за дезертира - из-за мощных ног и отсутствия ран на теле.
Назовите населенный пункт, из которого ОН прибыл в столицу.

Ответ:
Марафон.

Комментарий:
ОН - это воин Фидиппид, принесший в Афины весть о победе над персами при
Марафоне. В эссе Альгерда Бахаревича афиняне считают Фидиппида не
героем, а трусом, бежавшим с поля битвы.

Источник:
А. Бахаревич. Каляндар Бахарэвiча. - Мн.: Радыё Свабода, 2014.

Автор:
Сергей Дубелевич

Вопрос 2:
В конце одного детективного рассказа один из персонажей произносит
фразу: "ИКС правосудия на этот раз не помог, но его ИГРЕК по-прежнему
обязан карать". Назовите ИКС и ИГРЕК.

Ответ:
Щит, меч.

Зачет:
В любом порядке.

Источник:
А. Конан Дойл. Постоянный пациент. http://flibusta.is/b/214607/read

Автор:
Сергей Апанович

Вопрос 3:
Герой романа Джонатана Литтелла получает пулевое ранение в голову. После
долгой реабилитации он отмечает, что его восприятие мира значительно
улучшилось, и сравнивает дырку в своей голове с ЭТИМ. Одно из названий
ЭТОГО - Аджна. Назовите ЭТО двумя словами.

Ответ:
Третий глаз.

Комментарий:
"Мне казалось, что дырка у меня в голове стала третьим глазом, и этот
глаз не выносит ослепительного света". Аджна - термин из индуизма.

Источник:
   1. Дж. Литтелл. Благоволительницы. http://flibusta.is/b/266164/read
   2. https://ru.wikipedia.org/wiki/Чакра#Аджна_(третий_глаз)

Автор:
Сергей Апанович

Вопрос 4:
Проводя параллели между известными сюжетами, Дмитрий Быков сравнивает
Бывалого не со ВТОРЫМ, а с ПЕРВЫМ. В роли же ЧЕТВЕРТОГО писатель видит
Шурика. Назовите ЧЕТВЕРТОГО.

Ответ:
Д'Артаньян.

Комментарий:
Быков сравнивает Бывалого не с Портосом (хотя он обладает внушительной
комплекцией), а с флегматичным Атосом. Балбес - Портос, Трус - Арамис,
Шурик - гасконец.

Источник:
https://www.kommersant.ru/doc/2290603

Автор:
Сергей Дубелевич

Вопрос 5:
   <раздатка>
   Molekularkinetischen
   </раздатка>
   Блогер Дмитрий Чернышёв рассказывает, как один эскимос, не знавший
английский язык, долгое время перемещался по территории Соединенных
Штатов, просто повторяя на вокзале то, что говорил стоявший перед ним в
очереди человек. Чернышёв озаглавил пост об этом фразой "Ветераны ЕГО".
На раздаточном материале вы можете увидеть слово, которое встречается в
названии статьи Альберта Эйнштейна о НЕМ. Назовите ЕГО двумя словами.

Ответ:
Броуновское движение.

Комментарий:
Движение эскимоса было хаотичным, подобно движению броуновской частицы.
Полностью название статьи в русском переводе звучит так: "О движении
взвешенных в покоящейся жидкости частиц, требуемом
молекулярно-кинетической теорией теплоты".

Источник:
   1. https://mi3ch.livejournal.com/3909438.html
   2. https://onlinelibrary.wiley.com/doi/abs/10.1002/andp.19053220806
   3. https://fermlab.hse.ru/data/2013/06/09/1283651839/Einstein%28rus%29.pdf

Автор:
Илья Колодник

Вопрос 6:
В одном из своих стихотворений Борис Пастернак упоминает "птиц из породы
люблю вас". Согласно литературоведу Константину Поливанову, в этой
строке есть отсылка к немецкому языку, а также явно прослеживается связь
с музыкальным произведением XIX века. Назовите его автора.

Ответ:
[Петр Ильич] Чайковский.

Зачет:
[Камиль] Сен-Санс.

Комментарий:
"Люблю вас" по-немецки - "liebe dich", созвучно со словом "лебеди".
Музыкальное произведение, в котором есть лебеди, - это "Лебединое озеро"
Чайковского, однако вы могли подумать, что речь об "Умирающем лебеде"
Сен-Санса, что тоже не противоречит действительности.

Источник:
https://arzamas.academy/materials/393

Автор:
Команда "Легионеры Боливарии"

Вопрос 7:
   <раздатка>
   "Недостающее звено" - внушительный двухтомник, отвечающий на все
вопросы антропогенеза: <...> автор подробно и в меру иронично излагает
содержание предыдущих серий происхождения человека, рассказывает обо
всех возможных предках Homo Sapiens, объясняет, почему и как человек
родился именно в семействе гоминидов, и даже предсказывает наше будущее
как вида с точки зрения антропогенеза".
   </раздатка>
   В текст этой рецензии мы добавили две буквы к одному из слов.
Напишите эти две буквы.

Ответ:
Не.

Комментарий:
На самом деле книга антрополога Станислава ДробышЕвского называется
"Достающее звено" - но она действительно достаточно подробна, чтобы
оправдывать такое название.

Источник:
https://www.corpus.ru/products/drobyshevski-dostajushee-zveno-obezjany.htm

Автор:
Елена Шибут

Вопрос 8:
Живя в Париже в большой бедности, художник ХаИм СУтин часто работал в
полуобморочном состоянии. Дело в том, что любому приему пищи должен был
предшествовать ОН. Назовите ЕГО словом иностранного происхождения.

Ответ:
Натюрморт.

Комментарий:
Сутину нечасто удавалось добыть еду, но даже когда он это делал, то
всегда сначала рисовал с нее натюрморт, а уже потом принимался за еду.

Источник:
https://kulturologia.ru/blogs/241117/36766/

Автор:
Сергей Дубелевич

Вопрос 9:
Персонаж повести "Верный Руслан" называет своего соседа Трезорку
ничтожным псом - несмотря на то что ИХ у Трезорки вдвое больше, чем у
него самого. Назовите ИХ.

Ответ:
[Буквы] Р.

Комментарий:
Главный герой повести "Верный Руслан" - собака. Цитата: "Псом он
оказался совсем ничтожным, даром что кличку носил с двумя рокочущими
"Р", напоминающими рычание". В имени Трезорка букв "Р" вдвое больше, чем
в "Руслане".

Источник:
Г.Н. Владимов. Верный Руслан. http://flibusta.is/b/244572/read

Автор:
Сергей Дубелевич

Вопрос 10:
В 1982 году режиссер Питер Селларс высказал идею о создании оперы "ИКС в
Китае" по мотивам событий, случившихся за десять лет до того. По его
мнению, в этом сюжете был заложен огромный потенциал, базирующийся на
встрече Востока и Запада. В одном из слов вопроса мы пропустили
несколько букв. Воспроизведите его в исходном виде.

Ответ:
Никсон.

Комментарий:
Опера получила название "Никсон в Китае" и базировалась на реальных
исторических событиях.

Источник:
https://ru.wikipedia.org/wiki/Никсон_в_Китае

Автор:
Виталий Захарик

Вопрос 11:
Действие романа "Пентаграмма" происходит в крупном столичном городе.
Ответьте двумя словами: что автор сравнил с силуэтами жирафов,
высящимися на фоне закатного неба?

Ответ:
Башенные краны.

Зачет:
Строительные краны; подъемные краны; портовые краны.

Источник:
Ю Несбё. Пентаграмма. http://flibusta.is/b/164904/read

Автор:
Сергей Апанович

Вопрос 12:
   <раздатка>
   "Играют ли доги в домино? ПЕРВЫЙ и ВТОРОЙ в поисках единой теории
мироздания".
   </раздатка>
   Внимание, в вопросе есть замены.
   Книга Пола Хэлперна, посвященная двум известным ученым, в
русскоязычном варианте получила название "Играют ли доги в домино?
ПЕРВЫЙ и ВТОРОЙ в поисках единой теории мироздания". Догадавшись, кто
эти ПЕРВЫЙ и ВТОРОЙ, ответьте: какие слова мы заменили словами "доги" и
"домино"?

Ответ:
Коты, кости.

Комментарий:
Книга Пола Хэлперна называется "Играют ли коты в кости? Эйнштейн и
Шрёдингер в поисках единой теории мироздания" - название косвенно
строится на известных высказываниях этих физиков.

Источник:
https://www.piter.com/collection/pop-science/product/igrayut-li-koty-v-kosti-eynshteyn-i-shryodinger-v-poiskah-edinoy-teorii-mirozdaniya

Автор:
Елена Шибут

Вопрос 13:
Саркастичный Альфонс Алле приписал ИКСУ фразу "В любой жизни есть взлеты
и падения". А один футбольный журналист упоминает ИКСА в статье под
названием "Из грязи в князи и обратно". Какую профессию мы заменили
словом "ИКС"?

Ответ:
Лифтер.

Комментарий:
Футбольные специалисты термином "лифтер" называют команды, которые
регулярно вылетают в низшую лигу, в следующем сезоне возвращаются в
высшую и т.д.

Источник:
   1. https://ru.wikiquote.org/wiki/Альфонс_Алле
   2. https://footballhd.ru/articles/62765-fk-deportivo-iz-gryazi-v-knyazi-i-obratno.html

Автор:
Виталий Захарик

Вопрос 14:
В одном романе при рассказе о том, что семья героини разбогатела совсем
недавно, упоминается запах АЛЬФЫ. Назовите АЛЬФУ.

Ответ:
Типографская краска.

Зачет:
Краска.

Комментарий:
"Ее родители, простые крестьяне, разбогатели недавно, и их деньги
по-прежнему пахли типографской краской".

Источник:
Ю Несбё. Спаситель. http://flibusta.is/b/187899/read

Автор:
Сергей Апанович

Вопрос 15:
Участник передачи "Между молотом и наковальней" сравнил процесс закалки
оружия с ЭТИМ. По-гречески ЭТО - "баптИсма". Назовите ЭТО одним словом.

Ответ:
Крещение.

Комментарий:
Опустив в воду раскаленный кусок металла, кузнец словно "крестил"
клинок. В основе баптистского вероучения лежит принцип добровольного и
сознательного крещения по вере взрослых людей при наличии твердых
христианских убеждений и отказа от греховного образа жизни.

Источник:
Передача "Между молотом и наковальней" на телеканале "Viasat History",
выпуск от 13.12.2017 г.

Автор:
Сергей Апанович

Вопрос 16:
   <раздатка>
   То, что у всех есть роль своя, - не факт.
   Мир - не для нас построенная сцена.
   А жизнь - как затянувшийся антракт,
   Где после (ПРОПУСК 1) ждут (ПРОПУСК 2).
   </раздатка>
   В стихотворении Михаила БУчека пропущены фамилии современников.
Воспроизведите их.

Ответ:
Мендельсон, Шопен.

Комментарий:
Авторы свадебного и похоронного маршей. Мендельсон и Шопен жили в одно и
то же время (годы жизни - 1809-1847 и 1810-1849 соответственно).

Источник:
   1. https://www.stihi.ru/2015/05/11/1174/
   2. https://ru.wikipedia.org/wiki/Мендельсон,_Феликс
   3. https://ru.wikipedia.org/wiki/Шопен,_Фридерик

Автор:
Сергей Дубелевич

Вопрос 17:
Послушайте цитату известного американского режиссера: "Естественный,
свободный человек - это человек злой, деструктивный. Так называемое
"добро" - лишь способ, с помощью которого абсолютно отчужденное от
человека государство подчиняет людей, превращает их в...". Закончите
фразу двумя словами.

Ответ:
"... заводных апельсинов".

Комментарий:
Это цитата Стэнли Кубрика о фильме "Заводной апельсин".

Источник:
https://ru.wikiquote.org/wiki/Стэнли_Кубрик

Автор:
Виталий Захарик

Вопрос 18:
По одной из версий, ИКС был приобретен 2 апреля 1917 года в Нью-Йорке,
на Пятой авеню, в магазине сантехники. Окончательная судьба ИКСА
неизвестна - предполагают, что вскоре он был выброшен как мусор. Какое
слово с удвоенной согласной мы заменили словом "ИКС"?

Ответ:
Писсуар.

Комментарий:
Речь идет о знаменитом произведении Марселя Дюшана "Фонтан", основой
которого стал обычный фаянсовый писсуар, приобретенный в магазине
сантехники. Вскоре после демонстрации на выставке оригинал работы
пропал, по предположению биографа Дюшана - был выброшен. Желаем вам
провести перерыв с пользой!

Источник:
   1. У. Гомперц. Непонятное искусство. От Моне до Бэнкси.
http://flibusta.is/b/468862/read
   2. https://ru.wikipedia.org/wiki/Фонтан_(Дюшан)

Автор:
Елена Шибут

Вопрос 19:
Статья Александра Дмитриева, рассказывающая о качестве услуг, которые
предлагает пассажирам своих рейсов во время полетов компания "Аэрофлот",
называется "Сервис отличного уровня". В этом названии мы заменили два
слова, одно из которых - предлог. Напишите их.

Ответ:
На высоте.

Комментарий:
Название статьи "Сервис на высоте" намекает и на высокое качество
обслуживания, и на сферу деятельности компании "Аэрофлот".

Источник:
https://www.kommersant.ru/doc/3293956

Автор:
Виталий Захарик

Вопрос 20:
(pic: 20170886.jpg)
   Эта фотография была сделана в 1962 году. В каком городе?

Ответ:
Берлин.

Зачет:
Западный Берлин.

Комментарий:
Дети строят игрушечную стену.

Источник:
https://mi3ch.livejournal.com/3109840.html

Автор:
Сергей Апанович

Вопрос 21:
СинкопАле - потеря сознания, которая провоцируется резкими движениями
головы или ее длительным вынужденным необычным положением. Напишите
образованное от имени собственного прилагательное, которое присутствует
в названии одного из самых известных синкопАльных состояний.

Ответ:
Сикстинская.

Комментарий:
Так называемый "синдром Сикстинской капеллы" возникает при длительном
необычном положении головы - например, при рассматривании потолочной
росписи храмовых сооружений.

Источник:
А.С. Никифоров, Е.И. Гусев. Общая неврология.

Автор:
Климентий Комиссаров

Вопрос 22:
Во время гонки в рамках одного из этапов Кубка мира по биатлону
температура воздуха оказалась выше, чем ожидалось. В связи с этим
комментатор трансляции на телеканале "Матч-ТВ" пошутил, что ИКСА не
повысили в звании, видимо, намекая на то, что победы российским
спортсменам в этот день добиться не удастся. Назовите ИКСА двумя
словами, начинающимися на одну и ту же букву.

Ответ:
Майор Мороз.

Комментарий:
Россиянам издавна помогает побеждать Генерал Мороз. В день гонки
температура была не очень низкой, поэтому Мороз дотянул максимум до чина
майора.

Источник:
Трансляция гонки этапа Кубка мира по биатлону на телеканале Матч-ТВ,
30.11.2017 г.

Автор:
Виталий Захарик

Вопрос 23:
Рекламное агентство "FP7" [эф-пи сЕвен] разработало серию рекламных
плакатов, которые привлекают внимание к проблеме злоупотребления
лекарствами, позволяющими лишь на какое-то время снять боль. На одном из
принтов человек расслабляется, лёжа на таблетке болеутоляющего, словно
на диване. Со временем таблетка рассасывается, и человек начинает
паниковать, ведь ИКС - это АЛЬФА. Температура АЛЬФЫ варьируется от 500
до 1200 градусов по Цельсию. Назовите ИКС и АЛЬФУ.

Ответ:
Пол, лава.

Источник:
https://www.adsoftheworld.com/media/print/kkt_orthopedic_spine_centre_painkillers_lava

Автор:
Денис Валянский

Вопрос 24:
[Ведущему: отточия в цитате игнорировать.]
   Журналист Уилл Гомперц пишет: "Представьте себе картонную коробку.
Условно говоря, ее разбирают <...>, чтобы получилась плоскость,
позволяющая нам видеть все стороны сразу. Но при этом авторы хотят
сохранить ощущение трехмерности, чего плоский план дать не может.
Поэтому они мысленно обходят коробку со всех сторон, выбирая такие
ракурсы, которые дадут оптимальное представление об объекте, <...> и
определенным образом состыковывают плоскости - <...> так, чтобы ПРОПУСК
все-таки угадывался...". Назовите фамилию любого из двух авторов, о
которых идет речь.

Ответ:
Брак.

Зачет:
Пикассо.

Комментарий:
Пропущенное слово - "куб". Пабло Пикассо и Жорж Брак считаются
основателями кубизма. В приведенной цитате искусствовед Гомперц
объясняет, почему термин "кубизм" является не совсем верным.

Источник:
У. Гомперц. Непонятное искусство. От Моне до Бэнкси.
http://flibusta.is/b/468862/read

Автор:
Елена Шибут

Вопрос 25:
Блиц.
   1. Закончите тремя словами афоризм Ашота Наданяна: "Лучше сто раз
поехать к прадедушке, чем один раз...".
   2. Закончите одним словом афоризм Ашота Наданяна: "Эпидемия - это
когда туберкулезная палочка передается как ...".
   3. По словам Ашота Наданяна, "чужой ИКС как звонок будильника:
вставать не хочется, а надо". Назовите ИКС коротким словом.

Ответ:
   1. "... отправиться к праотцам".
   2. "... эстафетная".
   3. Гимн.

Источник:
https://ru.wikiquote.org/wiki/Ашот_Сергеевич_Наданян

Автор:
Виталий Захарик

Вопрос 26:
Писатель Эрнест Клайн сравнил ИКСА с пиццей без одного куска. ИКС
упоминается в экономическом термине, описывающем реакцию на попытку
поглощения, которая заключается в запуске компанией - мишенью программы
встречной покупки враждебной фирмы. Назовите ИКСА.

Ответ:
Pac-Man.

Зачет:
PacMan; Пакман; Пэкмэн; Пэкмен.

Комментарий:
Упомянутый термин - "защита пакмана".

Источник:
   1. Э. Клайн. Первому игроку приготовиться.
http://flibusta.is/b/503132/read
   2. http://vocable.ru/termin/zaschita-v-stile-pacman.html

Автор:
Сергей Апанович

Вопрос 27:
В качестве ИКСА на различных сайтах можно увидеть, в том числе,
антистрессовую игрушку, животных, пытающихся поймать свой хвост,
изображения атома или изображения Солнечной системы. Какую деталь
использует в качестве ИКСА сайт оружейного аукционного дома Рок Айленда?

Ответ:
Барабан [револьвера].

Комментарий:
В качестве индикатора загрузки часто используются различные вращающиеся
вещи - например, атом или животные, ловящие себя за хвост. Словом
"спиннер" называют не только антистрессовую вращающуюся игрушку, но и
сам индикатор загрузки.

Источник:
   1. https://www.google.ru/search?q=spinning+loading+indicator&tbm=isch
   2. https://www.rockislandauction.com/

Автор:
Команда "Легионеры Боливарии"

Вопрос 28:
В вопросе есть замена.
   Футбольный журналист, описывая игру ПОнтуса ВЕрнблума, который может
сыграть на разных позициях на поле, называет его УНИВЕРСАЛЬНЫМ СОЛДАТОМ.
Пик запросов в Google по словам "УНИВЕРСАЛЬНЫЙ СОЛДАТ" пришелся на
период с 13 по 19 августа 2017 года. Какие два слова мы заменили словами
"УНИВЕРСАЛЬНЫЙ СОЛДАТ"?

Ответ:
"Тысячеликий герой".

Комментарий:
Комментируя многозадачность Вернблума, журналист "Евроспорта" Дмитрий
Панфёров употребил название книги Джозефа Кэмпбелла "Тысячеликий герой".
Пик популярности запросов в Google объясняется тем, что во время своего
баттла эту книгу упомянул рэпер Оксимирон, чем вызвал небывалый рост ее
популярности. До 13 августа 2017 года - даты баттла среднее число
запросов в Google по фразе "Тысячеликий герой" было в среднем меньше
одного в день, после же резко возросло.

Источник:
   1. https://www.eurosport.ru/football/europa-league/2017-2018/story_sto6642889.shtml
   2. https://trends.google.com/trends/explore?q=тысячеликий%20герой

Автор:
Команда "Фунтики"

Вопрос 29:
В вопросе есть замены.
    Варлам Шаламов пишет, что мясо любых животных теряет свой
специфический запах, если его ПОВЕСИТЬ НА ГВОЗДЬ. ВЕШАНЬЕ НА ГВОЗДЬ -
прием, используемый при приготовлении таких блюд, как кивиАк и хАукарль.
Какое словосочетание мы заменили фразой "ПОВЕСИТЬ НА ГВОЗДЬ"?

Ответ:
Закопать в землю.

Зачет:
Зарыть в землю.

Комментарий:
Таким вот характерным для скандинавской кулинарии способом ссыльные в
ГУЛАГе готовили белок, ворон и других животных.

Источник:
   1. В.Т. Шаламов. Сухим пайком. http://flibusta.is/b/474339/read
   2. https://bigpicture.ru/?p=518846

Автор:
Сергей Апанович

Вопрос 30:
   <раздатка>
   Так было и так есть, так непременно будет,
   Пока не сдан проект, рассвет нас не разбудит.
   Пока не стихнет спор за окнами общаги,
   Пока ложится ПРОПУСК на белый лист бумаги.
   </раздатка>
   Перед вами отрывок из неофициального гимна Уральского
архитектурно-художественного университета. Заполните пропуск,
использовав два дефиса.

Ответ:
Koh-i-noor.

Зачет:
Кох-и-нур; Кох-и-нор; с незначительными грамматическими отклонениями.

Комментарий:
"Koh-i-noor" - знаменитая чешская марка канцтоваров, в первую очередь
карандашей.

Источник:
   1. http://club.mabuka.ru/content/gimn-uralgakha-planshetnaya-strana
   2. https://ru.wikipedia.org/wiki/Koh-i-Noor_Hardtmuth

Автор:
Виталий Захарик

Вопрос 31:
[Ведущему: отточие в цитате игнорировать.]
   Прослушайте цитату: "Когда вся тяжесть происходящего ложится на <...>
одного человека, он исчезает, как подломившаяся опора". Назовите
произведение, увидевшее свет в 1957 году, фрагмент из которого мы вам
привели.

Ответ:
"Атлант расправил плечи".

Зачет:
"Атлант расправил плечи. Часть II. Или - или".

Комментарий:
Смысл цитаты намекает на роль атланта.

Источник:
А. Рэнд. Атлант расправил плечи.
https://books.google.ru/books?id=o37NAhf9mBMC&pg=PA449#v=onepage&q&f=false

Автор:
Сергей Апанович

Вопрос 32:
Берлинская авангардная группа "Einstuerzende Neubauten" [айнштУрцэндэ
нойбАутн] известна тем, что для записи своих песен в числе прочего
извлекает звуки из объектов, найденных в городе. В тематическом
интернет-сообществе фотографию группы сопроводили цитатой из
произведения первой половины XX века. Назовите автора этого
произведения.

Ответ:
[Владимир] Маяковский.

Комментарий:
"А вы ноктюрн сыграть могли бы на флейте водосточных труб?" - вопрошал
Маяковский в стихотворении "А вы могли бы?". Среди "инструментов"
"Einstuerzende Neubauten" можно найти в том числе и водосточные трубы.

Источник:
(pic: 20170887.jpg)

Автор:
Команда "Легионеры Боливарии"

Вопрос 33:
Современный писатель называет свежеоштукатуренный фасад здания сырой
мечтой ИКСА. ИКСЫ являются главными героями фильма "БомбИ систему".
Назовите ИКСА.

Ответ:
Граффитист.

Зачет:
Граффитчик; художник граффити; уличный художник.

Комментарий:
Одно из значений слова "бомбить" - рисовать граффити.

Источник:
   1. Ю Несбё. Спаситель. http://flibusta.is/b/187899/read
   2. https://en.wikipedia.org/wiki/Bomb_the_System

Автор:
Сергей Апанович

Вопрос 34:
В стихотворении, посвященном своему знаменитому соотечественнику,
Евгений Евтушенко упоминает запретные меловые линии и сравнивает
адресата с поэтом, не побоявшимся "лезть в политику". Назовите этого
соотечественника.

Ответ:
[Лев] Яшин.

Комментарий:
Яшин стал первым известным вратарем, активно игравшим за пределами
штрафной. Евтушенко сравнивает Яшина с поэтами-шестидесятниками.

Источник:
Е.А. Евтушенко. Вратарь выходит из ворот.
http://retro-sport.sitecity.ru/ltext_0602180003.phtml?p_ident=ltext_0602180003.p_0602183141

Автор:
Сергей Дубелевич

Вопрос 35:
Вышедший из храма монах, персонаж Умберто Эко, принял ЭТО за огненный
пот. А вот Анджею Сапковскому ЭТО напомнила слеза, скатившаяся по щеке.
Назовите ЭТО двумя словами.

Ответ:
Капля воска.

Зачет:
Капли воска; восковая капля; восковые капли.

Комментарий:
Герой Умберто Эко выходил из храма, погруженный в богобоязненные мысли.
Капнувший с церковной свечи воск он сравнил с огненным потом.

Источник:
   1. У. Эко. Имя Розы.
https://books.google.ru/books?id=8-fRBQAAQBAJ&pg=PT209#v=onepage&q&f=false
   2. А. Сапковский. Меч Предназначения.
http://flibusta.is/b/442970/read

Автор:
Сергей Апанович

Вопрос 36:
По словам художника-абстракциониста Василия Кандинского, "АЛЬФА в нашем
представлении является теснейшей и единственной в своем роде связью
молчания и речи. Царство АЛЬФЫ беспредельно". Назовите АЛЬФУ одним
словом.

Ответ:
Точка.

Комментарий:
А мы ставим точку в нашем пакете.

Источник:
https://ru.wikiquote.org/wiki/Василий_Васильевич_Кандинский

Автор:
Виталий Захарик

Тур:
6 этап

Дата:
13-Apr-2018

Редактор:
1-18 - Василий Бобков и Анастасия Балмакова (Минск); 19-36 - Вера
Рабкина (Минск)

Инфо:
Василий Бобков и Анастасия Балмакова благодарят за тестирование и ценные
замечания соведущие команды, а также лично Веру Рабкину, Дмитрия
Медведева, Евгению Гуз, Надежду Потрихалину, Максима Мацуту, Елену
Фадееву, Анну Лущик, Александра Столярова, Алексея Блатуна, Михаила
Мальцева, Антона Ушкалова, Александра Марцинкевича и команду "Клёк"
(Минск). Вера Рабкина выражает благодарность командам "Одушевленные
аэросани", "Енотики-7", "И пусть никто не уйдет обиженный", "Эталон
этанола", "New-реанимация".

Вопрос 1:
Согласно научно-популярному фильму, Дарвин, путешествуя на корабле
"Бигль", понимал, что ответ на вопрос о происхождении жизни кроется
ПРОПУСК. Пропущенные слова входят в известное пожелание. Напишите
пропущенные слова.

Ответ:
Под килем.

Комментарий:
Дарвин пришел к убеждению, что жизнь зародилась в воде, т.е. буквально
под килем корабля. "Семь футов под килем" - известное пожелание удачи.

Источник:
   1. Документальный фильм Би-Би-Си "Океаны Солнечной системы".
   2. https://frazeolog_ru.academic.ru/433/

Автор:
Команда "Одушевленные аэросани"

Вопрос 2:
Сол Шульман пишет, что в начале XIX века в Австралии единственной ЕЮ
был, как ни странно, ром. Назовите ЕЕ двумя словами.

Ответ:
Твердая валюта.

Комментарий:
Забавно, что жидкий ром был твердой валютой.

Источник:
С. Шульман. Австралия - Terra Incognita: Когда звери еще были людьми.
http://flibusta.is/b/312229/read

Автор:
Команда "Одушевленные аэросани"

Вопрос 3:
Увлечение рыбалкой стало поводом ЕГО знакомства с Фиделем Кастро,
которому ОН вручал приз за победу в рыболовных соревнованиях. Назовите
ЕГО.

Ответ:
[Эрнест] Хемингуэй.

Комментарий:
Эрнест Хемингуэй увлекался рыбалкой и долгое время прожил на Кубе.

Источник:
https://ru.wikipedia.org/wiki/Дом-музей_Эрнеста_Хемингуэя

Автор:
Василий Бобков

Вопрос 4:
Одной из причин появления легенд о НИХ являются пузырьки метана, которые
вибрируют на частоте инфразвука и могут вызывать у человека панику или
страх. Назовите ИХ словом, пишущимся через дефис.

Ответ:
Корабли-призраки.

Комментарий:
Одна из гипотез гласит, что в районе Бермудского треугольника скопилось
огромное количество метана из-за когда-то действовавших на дне вулканов.
Температура и давление воды выталкивают газ на поверхность. Люди в
страхе и панике просто прыгали за борт и погибали.

Источник:
https://www.eg.ru/science/479115/

Автор:
Василий Бобков

Вопрос 5:
(pic: 20170888.jpg)
   Перед вами портрет, на котором изображен Эдвард Джеймс. Назовите
автора этого портрета.

Ответ:
Рене Магритт.

Комментарий:
Любил Магритт вместо головы рисовать что-то другое.

Источник:
http://www.vokrugsveta.ru/article/283705/

Автор:
Василий Бобков

Вопрос 6:
В книге Андрея Фатющенко "Семь чудес света" можно прочесть истории,
рассказанные жителями разных стран. Какое слово мы пропустили в
предыдущем предложении?

Ответ:
Миллиардов.

Комментарий:
Книга называется "Семь миллиардов чудес света" и рассказывает о том,
насколько каждый человек важен и уникален.

Источник:
https://www.ozon.ru/context/detail/id/4794596/

Автор:
Анастасия Балмакова

Вопрос 7:
Стивен Спилберг с сожалением отмечает, что при общении с современной
молодежью разговор занимает пять минут, а ОНА гаджету - двадцать.
Благодаря "ЕЙ" в 2008 году одно европейское государство смогло
заработать на туристах. Назовите ЕЕ.

Ответ:
Молитва.

Комментарий:
Стивен Спилберг не одобряет всеобщую погруженность в телефоны и
сравнивает позу молодых людей с молящимся человеком. Песня "Молитва"
певицы Марии Шерифович позволила Сербии принять конкурс "Евровидение".

Источник:
   1. http://readrate.com/rus/news/pervomu-igroku-prigotovitsya-fantastika-osnovannaya-na-realnosti
   2. https://www.obozrevatel.com/finance/analytics-and-forecasts/72138-evrovidenie-2017-za-chto-boryutsya-5-gorodov-kandidatov-dyira-v-byudzhete-ili-ryivok-dlya-turizma.htm

Автор:
Дарья Соловей

Вопрос 8:
(pic: 20170889.jpg)
   Александр Пиперски отмечает, что коренные обитатели планеты Пандора
используют ЕЕ. Назовите ЕЕ.

Ответ:
Восьмеричная система [счисления].

Комментарий:
У цивилизации, которая пользуется языком на'ви, на руках по четыре
пальца, а не по пять, и им удобно двумя руками досчитать до восьми.

Источник:
https://postnauka.ru/video/77568

Автор:
Команда "Одушевленные аэросани"

Вопрос 9:
Проблему "искусственного полета" удалось успешно решить лишь после того,
как братья Райт перестали имитировать птиц и приступили к изучению
аэродинамики. Возможно, именно этим объясняется непопулярность одной
известной задачи среди ученых. Назовите эту задачу двумя словами,
начинающимися на одну и ту же букву.

Ответ:
Тест Тьюринга.

Комментарий:
Исследователи практически не занимаются решением задачи прохождения
теста Тьюринга, считая, что гораздо важнее изучить основополагающие
принципы интеллекта, чем продублировать одного из носителей
естественного интеллекта.

Источник:
https://ru.wikipedia.org/wiki/Тест_Тьюринга

Автор:
Сергей Матмусаев

Вопрос 10:
Название статьи Александра Зотина о новой гипотетической форме
капитализма, при которой доля трудовых доходов упадет до нуля за счет
полной автоматизации, лишь одной буквой отличается от реально
существующего термина из двух слов. Воспроизведите название этой статьи.

Ответ:
"Робовладельческий строй".

Комментарий:
В статье рассказывается о том, что доля трудовых доходов будет
стремиться к нулю, а доля доходов от капитала, наоборот, приблизится к
100%. Всю работу станут делать роботы, а большинству людей придется
сидеть на пособии.

Источник:
https://www.kommersant.ru/doc/3455179

Автор:
Александр Мартинович

Вопрос 11:
Антон Заруцкий пишет, что по структуре перламутр напоминает ЕГО:
арагонит чередуется с хитином, который, просачиваясь сквозь поры в
пластинках, образует узкие мостики. Назовите ЕГО, использовав имя
собственное.

Ответ:
Торт "Наполеон".

Комментарий:
Перламутр устроен слоями.

Источник:
http://www.vokrugsveta.ru/article/279998/

Автор:
Василий Бобков

Вопрос 12:
В статье журнала "National Geographic" [нэшнл джиогрЭфик] приводится
сравнение с мухой, пролетающей мимо включенной фары автомобиля: яркость
меняется, и тогда можно обнаружить ЕЕ. Назовите ЕЕ словом греческого
происхождения.

Ответ:
Планета.

Зачет:
Экзопланета.

Комментарий:
Для распознавания планет телескоп отслеживает изменение яркости звезд.
Оно происходит, когда планета проходит по орбите вокруг звезды и
преграждает путь свету, который от нее исходит. Если телескоп фиксирует,
что затемнение происходит с определенной периодичностью, значит,
вероятнее всего, он обнаружил планету.

Источник:
http://www.nat-geo.ru/universe/476151-novaya-zemlya-nasa-obnaruzhilo-pervuyu-ekzoplanetu-pokhozhuyu-na-nashu/

Автор:
Дарья Соловей

Вопрос 13:
В этом вопросе слово "ИКС" является заменой.
   В 1891 году в штат Огайо прибыл Фрэнк Мельбурн, которого называли
"волшебником" и "заклинателем". Вскоре после этого местные фермеры
договорились вскладчину купить у Мельбурна один ИКС. Рекордное
количество ИКСОВ наблюдается на горе ВаиАлеале на Гавайях. Назовите ИКС
двумя словами, которые начинаются на одну и ту же букву.

Ответ:
Дождливый день.

Зачет:
День дождя.

Комментарий:
Мельбурн был бродячим заклинателем дождя. Фермеры Огайо страдали от
засухи и решили прибегнуть к его услугам. Мельбурну повезло - на
следующий день после его манипуляций действительно пошел дождь. На
склонах горы Ваиалеале на Гавайях наблюдается около 350 дождливых дней в
год, что является мировым рекордом.

Источник:
   1. С. Барнетт. Занимательное дождеведение: дождь в истории, науке и
искусстве. http://flibusta.is/b/460154/read
   2. http://odogde.ru/interesnoe-o-dozhde/samye-dozhdlivye-mesta-i-goroda-mira.html

Автор:
Команда "Одушевленные аэросани"

Вопрос 14:
(pic: 20170890.jpg)
   Перед вами кадр из фильма "Назад в будущее - 3". Забавно, что в 1885
году ИХ еще не существовало. Назовите ИХ. Принимается описательный
ответ.

Ответ:
Шрифты на надгробии.

Зачет:
Шрифт Helvetica, шрифт EuroStile, а также описательные ответы с
упоминанием слова "шрифт" или "гарнитура". Незачет: Шрифты с засечками,
шрифты без засечек, курсив.

Комментарий:
Шрифты Helvetica и EuroStile появились значительно позже (в 1957 и 1962
годах соответственно). Забавно, что и здесь наблюдается в некотором
смысле путешествие [шрифта] во времени.

Источник:
   1. Нетология: лекция "Основы графического дизайна: композиция, цвет,
типографика".
   2. Х/ф "Назад в будущее - 3" (1990), реж. Роберт Земекис.
   3. https://ru.wikipedia.org/wiki/Гельветика
   4. https://en.wikipedia.org/wiki/Eurostile

Автор:
Анастасия Балмакова

Вопрос 15:
[Ведущему: сделать небольшую паузу после слова "лавы".]
   При излиянии темной жидкой лавы более легкий и светлый кварц
"выдавливается" из породы и застывает в трещинах. Ричард Форти
сравнивает кварцевую массу с НЕЙ. Назовите ЕЕ.

Ответ:
СУкровица.

Зачет:
Лимфа.

Комментарий:
Кварцевая масса застывает в трещинах породы подобно сукровице в ране.
Кровь вытекает из раны, лава - из Земли. Такая вот анатомическая
аналогия.

Источник:
Р. Форти. Трилобиты: свидетели эволюции.
http://flibusta.is/b/363864/read

Автор:
Анастасия Балмакова

Вопрос 16:
Тираннозавры жили в позднем меловом периоде, в длину они достигали 35-40
метров, а весили до 110 тонн. Какое слово мы немного изменили в
предыдущем предложении?

Ответ:
Титанозавры.

Комментарий:
Тираннозавры, конечно, были страшными, но их масса была всего около 6
тонн. Гигантские же стотонные динозавры за свой размер получили название
титанозавры.

Источник:
https://ru.wikipedia.org/wiki/Титанозавры

Автор:
Анастасия Балмакова

Вопрос 17:
   <раздатка>
   Alakshak
   </раздатка>
   Розданное вам слово с алеутского переводится как "великие зЕмли", или
ОН. Назовите ЕГО двухкоренным словом.

Ответ:
Полуостров.

Комментарий:
Розданное вам слово означает Аляску. Аляска - полуостров на
северо-западе Северной Америки.

Источник:
https://ibigdan.livejournal.com/14828195.html

Автор:
Василий Бобков

Вопрос 18:
Памятник-фонтан в саду музея в районе Мейлахти представляет собой ЕГО.
Ольга Воробьёва называет ЕГО "желтобрюхим божеством". Назовите ЕГО.

Ответ:
Самовар.

Комментарий:
Памятник-фонтан находится в Хельсинки, и здесь, кроме столика с
самоваром и заварочным чайником, есть еще две чашки с блюдцами и
подсвечник. Две струи воды, из заварочного чайника на самоваре и из
краника самовара, льются прямо на стол.

Источник:
https://www.nkj.ru/open/32614/

Автор:
Василий Бобков

Вопрос 19:
При въезде в какой европейский город вас встречает плакат, текст
которого можно перевести как "Приятно познакомиться!"?

Ответ:
Ницца.

Источник:
ЛНА.

Автор:
Дмитрий Медведев

Вопрос 20:
Герой Роджера Желязны сравнивает медсестру с АЛЬФОЙ, но замечает, что
отличие в том, что все медсестры похожи друг на друга. Какое слово мы
заменили АЛЬФОЙ?

Ответ:
Снежинка.

Комментарий:
Как известно, все снежинки разные.

Источник:
Р. Желязны. Знак Единорога. http://flibusta.is/b/139509/read

Автор:
Никита Геер

Вопрос 21:
Штат Невада пустынен и беден фауной, поэтому в конце 2016 года в
качестве НЕГО был выбран рыцарь. Назовите ЕГО.

Ответ:
Название [хоккейной] команды.

Зачет:
Маскот/талисман/логотип [хоккейной] команды; маскот/талисман/логотип.

Комментарий:
В конце 2016 года в НХЛ добавился 31-й клуб - из Лас-Вегаса. Из-за
скудности выбора команду пришлось назвать не в честь животных, а в честь
рыцарей.

Источник:
https://www.americansport.ru/las-vegas-golden-knights-komanda-nhl/

Автор:
Павел Новиков (Витебск)

Вопрос 22:
Профессор Карл Кокелл разработал тартан для ношения во время
определенной научно-исследовательской деятельности. Синий цвет на
тартане символизирует наличие воды в прошлом, две толстые белые линии -
это полюсы. А что символизирует красный фон?

Ответ:
Поверхность Марса.

Зачет:
Марс.

Источник:
https://mi3ch.livejournal.com/3799668.html

Автор:
Надежда Потрихалина

Вопрос 23:
Клоч - это главное блюдо зороастрийского праздника Сарсал, отмечающегося
весной. Ответьте двумя словами: что принято разбрасывать по полям и
огородам в этот день для повышения плодородности?

Ответ:
Яичная скорлупа.

Зачет:
Яичные очистки; скорлупа яиц.

Комментарий:
"Клоч" крайне созвучен со словом "кулич", у Сарсала и Пасхи много схожих
черт: пироги-куличи, крашеные яйца, празднование весной. Также известно,
что яичная скорлупа является хорошим удобрением, так что неудивительно,
что обряд эффективен.

Источник:
https://kulturologia.ru/blogs/070418/38512/

Автор:
Вера Рабкина

Вопрос 24:
После смерти Маяковского распространился слух, будто поэт был убит.
Появлению слуха способствовал Константин Луцкий, который по неопытности
использовал недостаточное количество вазелина при изготовлении ЕЕ.
Назовите ЕЕ двумя словами.

Ответ:
Посмертная маска.

Комментарий:
Скульптор Константин Луцкий снимал с Маяковского посмертную маску, но
плохо смазал лицо вазелином, сорвал кожу со щеки и переносицы, отчего
возник слух, будто к Маяковскому был подослан убийца, поэт сопротивлялся
и в драке получил рассечение щеки.

Источник:
Д.Л. Быков. Тринадцатый апостол. Маяковский. Трагедия-буфф в шести
действиях. http://flibusta.is/b/450896/read

Автор:
Команда "ИПННУО"

Вопрос 25:
Роман "Толстая тетрадь" рассказывает о судьбе двух близнецов,
разлученных в детстве. Их судьбы хоть и похожи, но перемешались совсем
разным образом. Одного из братьев зовут Лукас. Как зовут второго?

Ответ:
Клаус.

Комментарий:
Их имена являются анаграммами.

Источник:
А. Кристоф. Толстая тетрадь. http://flibusta.is/b/144373/read

Автор:
Вера Рабкина

Вопрос 26:
Из-за близости к буддизму на ИХ форме начала XX века советская звезда
соседствовала с желтой свастикой, что впоследствии привело к репрессиям.
Рассказывают, что в 1949 году при отмечании юбилея даже решили опустить
некоторые строки произведения. Назовите это произведение.

Ответ:
"Памятник".

Зачет:
"Я памятник себе воздвиг нерукотворный...".

Комментарий:
Речь идет о калмыках. Когда отмечался 150-летний юбилей Пушкина, во
время публичного чтения по радио стихотворения "Памятник" было опущено
часть строк, чтобы избежать произнесения слов "... и друг степей
калмык".

Источник:
https://kulturologia.ru/blogs/080418/38529/

Автор:
Вера Рабкина

Вопрос 27:
Фредрик Шёберг рассказывает о местах, где развитие фауны и флоры пошло с
нуля. Как пример он приводит остров Суртсей вблизи Исландии, а также
упоминает другой остров в южном полушарии. Какой?

Ответ:
Кракатау.

Комментарий:
"... на острове вулканического происхождения Суртсей, выступившем из
моря около Исландии. Или на Кракатау в Зондском проливе, между островами
Ява и Суматра, где в 1883 году взорвался вулкан, вследствие чего дальше
развитие и фауны и флоры пошло с нуля".

Источник:
   1. Ф. Шёберг. Ловушка Малеза, или О счастье жить в плену необычной
страсти, мухах и причудах судьбы. http://flibusta.is/b/442189/read
   2. https://ru.wikipedia.org/wiki/Кракатау

Автор:
Никита Геер

Вопрос 28:
Автор вопроса, надеясь провести время в тишине, всегда при покупке
билета на поезд выбирает одни и те же место и вагон. Какие?

Ответ:
4 33.

Зачет:
33 4.

Комментарий:
Очевидно, автор вопроса считает, что есть связь между местом в поезде и
сочинением американского композитора Джона Кейджа 4'33", на всём
протяжении которого участники ансамбля не извлекают звуков из своих
инструментов.

Источник:
   1. ЛНА.
   2. https://ru.wikipedia.org/wiki/4%E2%80%B233%E2%80%B3

Автор:
Евгения Гуз

Вопрос 29:
Шелковая АЛЬФА является обязательным атрибутом калмыцкого головного
убора. Другая АЛЬФА, согласно легенде, помогла определить, кому
принадлежит главенство над землями. Назовите АЛЬФУ двумя словами,
начинающимися на одну и ту же букву.

Ответ:
Красная кисть.

Комментарий:
Главенство над Ольстером было решено в результате состязания - нужно
было пересечь водный поток и первым коснуться рукой берега; для победы
один из состязавшихся отрезал себе кисть и кинул ее на берег,
впоследствии красная кисть Ольстера стала геральдическим символом.

Источник:
   1. https://kulturologia.ru/blogs/080418/38529/
   2. https://ru.wikipedia.org/wiki/Красная_рука_Ольстера

Автор:
???

Вопрос 30:
В ходе изучения системы СКЛАДЫВАНИЯ крыльев божьей коровки, ученые
ТОКИЙСКОГО университета пришли к выводу, что крылья имеют много общего с
ЭТИМ. Назовите ЭТО заимствованным словом.

Ответ:
Оригами.

Комментарий:
Компактность и способ складывания крыльев напомнило японцам искусство их
страны - оригами.

Источник:
https://www.sciencedaily.com/releases/2017/05/170515150714.htm

Автор:
Вера Рабкина

Вопрос 31:
В белорусском языке это заболевание носит название "прАнцы", что связано
с вражескими войсками, которые его завезли. Назовите это заболевание
по-русски.

Ответ:
Сифилис.

Комментарий:
Слово "пранцы" происходит от слова "Франция". Сифилис также называют
"французской болезнью", что нашло отражение и в белорусском языке.

Источник:
???

Автор:
Надежда Потрихалина

Вопрос 32:
"Жизнь после людей" - это научно-популярный фильм, который рассказывает,
что будет с планетой после исчезновения человечества. Какой город
приводится в качестве иллюстрации мира через двадцать лет после
исчезновения людей?

Ответ:
Припять.

Источник:
https://ru.wikipedia.org/wiki/Жизнь_после_людей

Автор:
Команда "ИПННУО"

Вопрос 33:
В книге "Вот я" Джонатан Фоер пишет, что иноземные садоводы подрЕзали
ЕГО Исаака до самой галисийской почвы. Назовите ЕГО двумя словами.

Ответ:
Семейное древо.

Зачет:
Генеалогическое древо.

Комментарий:
Садоводы были немецкие. Так Фоер иносказательно описывает Холокост.

Источник:
Дж.С. Фоер. Вот я. http://flibusta.is/b/507480/read

Автор:
Вера Рабкина

Вопрос 34:
   <раздатка>
   WAM
   </раздатка>
   В сериале о классической музыке над вариантом ИКСА работает робот под
кодовым именем WAM [даблъю-эй-эм]. Назовите ИКС одним словом.

Ответ:
Реквием.

Комментарий:
В сериале робот, проанализировав все работы Моцарта, пытается закончить
Реквием так, как его закончил бы сам мастер. Сам же робот получил
название по инициалам композитора - Вольфганг Амадей Моцарт.

Источник:
Телесериал "Моцарт в джунглях", s04e06.

Автор:
Вера Рабкина

Вопрос 35:
Чтобы обеспечить поселение Рьюкан достаточным ИКСОМ, на вершине горы
возле него установили огромные объекты, изменяющие положение каждые 10
секунд. Мы не спрашиваем, что это за объекты. Назовите ИКС двумя
словами.

Ответ:
Солнечный свет.

Комментарий:
Поселение расположено в долине таким образом, что почти полгода до него
не достают солнечные лучи. На вершине горы поставили сразу три огромных
зеркала, которые меняют свое положение каждые 10 секунд, следуя за
лучами солнца. Отраженный свет освещает примерно 600 квадратных метров,
что полностью покрывает центральную площадь городка.

Источник:
https://kulturologia.ru/blogs/090418/38536/

Автор:
Вера Рабкина

Вопрос 36:
Комментируя чрезмерную активность советской цензуры, персонаж Шамиля
Идиатуллина шутит, что скоро все иностранные фильмы станут ТАКИМИ.
Ответьте, используя слово с двумя корнями: какими - ТАКИМИ?

Ответ:
Короткометражными.

Комментарий:
Советская власть вырезАла из иностранных фильмов неугодные куски, из-за
чего фильмы всё больше и больше сокращались.

Источник:
Ш.Ш. Идиатуллин. Город Брежнев. http://flibusta.is/b/477611/read

Автор:
Вера Рабкина


FreeBSD-CVSweb <freebsd-cvsweb@FreeBSD.org>